Torts

¡Supera tus tareas y exámenes ahora con Quizwiz!

Arthur, a member of a teenage gang, fired a gun from a car during a drive-by shooting. Arthur intended the bullet to strike Bazelon, but instead it accidentally struck Clark. The best reason for Clark's attorney to frame the case against Arthur as a suit for ordinary negligence, rather than a suit for battery, is that: A. a judgment for negligence is more likely to be covered by insurance than one for battery. B. the doctrine of transferred intent, as traditionally formulated by the courts and the Restatement, is inapplicable to this set of facts. C. a judgment for negligence is more likely to support award of punitive damages than one for battery. D. a judgment for negligence is more likely than one for battery to be non-dischargeable in bankruptcy.

A

A minor child pulled a chair from behind a lady for the purpose of causing her to fall to the ground. As a result of the fall, the lady suffered serious injuries amounting to $20,000 in damages. The lady was in no way at fault with respect to her injuries. A statute in the jurisdiction provides: (1) In addition to any other remedy provided by law, the parent or parents of an unemancipated minor child shall be liable for actual damages to person or property caused by any tort intentionally or recklessly committed by such child. However, a parent who is not entitled to legal custody of the minor child at the time of the intentional or reckless tort shall not be liable for such damages. (2) The legal obligation of the parent or parents of an unemancipated minor child to pay damages under this section shall be limited to not more than $7,500, payable to the same claimant, for one or more acts. Aside from this statute, the jurisdiction adheres to the common-law rules on parental liability. The minor child lives with the child's mother and depends on her for support. One the stated facts, if the lady sues the child's mother, the most likely result is judgment for: A. the lady in the amount of $7500, regardless of whether mother had reason to foresee that her child would move the chair. B. the lady in the amount of $7500, but only if mother could foresee that her child would move the chair. C. the lady in the amount of $20,000, regardless of whether mother could foresee that her child would move the chair. D. for mother, if the she had no reason to see that her child would move the chair and cause harm.

A

An action based on negligence is less likely than an action based on intentional tort to: A. Give rise to punitive damages. B. Be covered by liability insurance. C. To be dischargeable in bankruptcy D. Give rise to respondeat superior liability

A

Assume that an insurance policy has a liability limits in the amount of $10,000. In which of the following cases will the liability insurer be least likely to be held liable for amounts in excess of policy limits: A. The insurer fails to accept an offer to settle the case for $8,000 prior to trial at a point when experts would say that there is a 90% chance that the defendant will be held liable for $50,000 and virtually no chance the verdict will be less than $30,000; at trial the defendant is held liable for $60,000. B. The insurer fails to accept an offer to settle the case for $9,500 prior to trial at a point when experts would say that there is a 90% chance that the verdict will not exceed $15,000 and only a 20% chance of liability; at trial the defendant is held liable for $20,000. C. The insurer fails to accept an offer to settle the case for $4,000 prior to trial at a point when experts would say that there is a 90% chance that the defendant will be held liable for between $500,000 and $700,000; at trial the defendant was held liable for only $12,000. D. None of the above, because an insurer cannot be held liable for amounts in excess of policy limits in the absence of evidence of fraud on the part of the insurer, and here there is no such evidence.

B

Blanche slipped on a grape in a grocery store and fell, injuring her leg. As a result of the fall, Blanche incurred a $30,000 in medical expenses. The jurisdiction has not adopted comparative negligence or comparative fault, and prior to trial it was highly likely that the jury would find that carelessness on Blanche's part contributed to the accident. The grocery store has an insurance policy providing $20,000 in coverage for accidents on the premises. Blanches attorney demanded the full policy limits to settle the case. The insurance company refused, and the case proceeded to trial. At trial, a crucial witness as to Blanche's carelessness was unavailable because of illness. As a result, the jury returned a verdict against the grocery store for $75,000 that Blanche was in no way responsible for the accident. To satisfy the judgment, the insurance company then pays $20,000 and the grocery store pays $55,000. The grocery store then sues the insurance company based on its failure to settle the case prior to trial. The most likely result is judgment for: A. the grocery store, because the judgment in the personal injury trial exceeded policy limits. B. the insurance company, because it was not unreasonable to decline to settle a case in which it was "highly likely" that the plaintiff would be found to have acted carelessly. C. the grocery store, because the amount of the settlement demand did not exceed policy limits. D. the insurance company, because its ultimate liability cannot exceed policy limits.

B

Over the years, a sidewalk near a law school building had sunk into the ground, so that the bottom step became unusually deep. A student who knows this fact regularly uses that entrance, even though other entrances to the building are available. One day the law student forgot about the dangerous bottom step, failed to take precautions, fell, and was injured. Assume that the student can prove that the school was negligent and that a jury will find that the accident was 65% the fault of the school and 35% the fault of the student, because the student failed to exercise care when leaving the building. In a jurisdiction adhering to modified comparative fault, the student can recover what part of the student's losses: A. Zero B. 65% C. 35% D. 30% E. 15% Choice

B

Jack purchased a bottle of AAA Champagne from the BBB Liquor Mart. The champagne was bottled by AAA Inc. utilizing bottles manufactured by the CCC Glass Co. While Jack was opening the bottle it suddenly exploded, causing disfiguring cuts to his face. Expert testimony establishes that the weakness in the structure of the glass bottle resulted from an externally caused crack in the bottom of the bottle and that the most likely source of such a crack is customer abuse of the bottles on display at the BBB Liquor Mart. BBB is a self-service store in which customers have free access to the merchandise. BBB attempts to monitor customer behavior but admits that it is impossible to assure that customers will not damage bottles. There is no reasonable alternative design available for the bottle that would prevent such accidents. If Jack sues AAA, BBB, and CCC seeking to impose strict liability for a product defect under Restatement, Third, of Torts: Products Liability, the most likely result is judgment in favor of Jack against: A. AAA only. B. BBB only. C. CCC only. D. AAA, BBB, and CCC. E. Neither AAA, BBB, nor CCC.

B is the correct answer.

Sachel owns a building. He previously used it for his residence for twenty years. Since moving to England three years ago, Sachel has rented the building for use as a shop. The turnover rate has been high. Sachel has had three different tenants. The first two, Aaron and Beto, left at the conclusion of their successive one-year leases, and the third, Carmen, is thinking about leaving when her one-year lease expires. For all three tenants, Sachel used the same written lease form. The document grants the tenant the right to exclusive possession and control of the property and requires the tenant to keep the property in good repair. During the last three years Sachel has not visited the property, has not been asked to make any repairs, and has not been told that any repairs have been made. Yesterday, Palmer, a pedestrian walking outside the building, was struck and injured when an awning hanging over the entrance pulled loose from the face of the building and fell to the ground. In a lawsuit based on the accident, an expert will testify that it is obvious from the debris that the apparatus attaching the awning to the building was defectively repaired on one or more occasions during the past five years, but that it is impossible to say exactly when the repairs were improperly performed. There is no other evidence relating to those issues. If Palmer sues Sachel for negligence and seeks to assert a res ipsa loquitur theory of liability, which of the following factors, if supported by the facts, is least relevant to (i.e., least useful in) determining whether res ipsa loquitur is applicable: A. During the last three years, the three tenants, rather than Sachel, have exercised exclusive control over the property. B. At all times during the past five years, Sachel owned the building. C. The lease document does not require a tenant to notify Sachel of repairs that they make and Sachel has not been informed of any such repairs during the past three years. D. There is no clear evidence as to exactly when the defective repairs more likely than not occurred.

B is the correct answer. Whether Sachel owned the building says little or nothing about whether Sachel was responsible for making the negligent repairs or aware of the fact relating to those repairs.

Darla fires a gun and the bullet strikes Pedro. Darla is most likely to escape liability for battery by introducing credible evidence that: A. She intended merely to scare Pedro and did not want to hit him. B. She was legally insane at the time of the shooting and her desire to shoot Pedro was caused by the insanity. C. She did not intend to point or fire the gun at Pedro, but accidentally pulled the trigger while examining the weapon. D. She believed that Pedro was wearing a bullet-proof vest that would deflect the bullet.

C

A nuclear power plant operated by Consolidated, Inc., malfunctions and leaks radiation into the community. As a result of the accident, Titus develops an unusual form of cancer which doctors will testify, and a jury will find, was caused solely by the leaked radiation. If Titus sues Consolidated to recover damages related to the cancer, the court will most likely enter judgment for: A. Titus, regardless of whether Consolidated failed to exercise care. B. Consolidated, if it proves that it exercised reasonable care in operation of the power plant. C. Titus, but only if other persons in the community suffered similar harm. D. Consolidated, if the variety of cancer that Titus developed is rare.

Choice A is correct.

A manufactures and packs a can of beans, which he sells to B, a wholesaler. B sells the beans to C, a jobber (that is, a second wholesaler), who resells it to D, a retail grocer. E buys the can of beans from D, and gives it to F. F serves the beans at lunch to G, his guest. While eating the beans, G breaks a tooth, on a pebble of the size, shape, and color of a bean, which no reasonable inspection could possibly have discovered. There is satisfactory evidence that the pebble was in the can of beans when it was opened. Under Restatement, Second, of Torts §402A, which parties can be held liable to G without proof of negligence? A. A, B, C, D, E, and F. B. Neither A, B, C, D, E, or F. C. A, B, C, and D only. C. A only. D. A and D only.

C is the correct answer.

Rocko's friend, Tyrone, was seriously injured by a piece of heavy machinery and was in need of immediate medical attention. While rushing Tyrone to the hospital in his car, Rocko caused an auto accident while traveling 78 m.p.h. in a 65 m.p.h. zone As result of the collision, Botley's truck was demolished. If Botley sues Rocko for negligence, and the jury finds that Rocko had a valid excuse for exceeding the speed limit, the violation of the state speed limit law will be: A. Conclusive evidence of Rocko's negligence. B. Some evidence that Rocko was negligent. C. No evidence that Rocko was negligent. D. A complete bar to recovery by Botley because there was an excuse for the violation.

C is the correct answer.

Alison keeps a young lion in her backyard to scare off any intruders. The lion is fastened to a post by a chain. Because of a defect in the chain that Alison could not have been expected to detect, the lion broke free and attacked Patrick on a nearby sidewalk outside the property, injuring him. Patrick sues Alison to recover damages for his injuries. The court will likely enter judgment for: (A) Patrick, regardless of whether Alison exercised reasonable care. (B) Patrick, but only if Alison failed to exercise reasonable care. (C) Alison, because the injuries were inflicted outside Alison's premises. (D) Alison, because Patrick was not an invitee.

Choice A is correct.

Sam, a man running and trying to board a moving train, is negligently pushed and pulled by two trainmen trying to "help" him to get on board. As a result of the conduct of the trainman, Sam dropped a package that he was carrying. The package was wrapped in newspaper and tied with string, so its contents were unknown. As a result, the crystal glassware contained in the package shattered when the package hit the ground. Sam seeks to recover damages for the destruction of the crystal. If he sues the railroad for which the trainmen work and proves that the trainmen were negligent, Sam will: A. Prevail, if the jurisdiction adheres to pure comparative negligence. B. Not prevail, because the trainmen did not know what was in the package. C. Not prevail, because the alleged tort did not occur within the trainmen's scope of employment. D. Not prevail, because Sam assumed the risk by running for the train.

Choice A

Acme manufactures glass beads and displays its products in a showroom where there are many bins containing large quantities of beads. Customers can reach into the bins and remove beads for closer inspection. While touring the showroom, Delphia slips and falls on a glass bead that was on the floor. In a negligence action against Acme to recover for her injuries, must Delphia prove that the Acme was aware of the bead that was on the floor that caused her to slip? A. No, because this type of risk was foreseeable from Acme's manner of operation. B. Yes, because a person cannot be held liable for harm caused by an allegedly dangerous condition absent actual notice of the condition. C. No, because on these facts there is evidence of constructive notice. D. No, because notice is not an element of a cause of action for negligence.

Choice A is correct

April, a young girl, has been forbidden to play with the dog that is owned by the Marx family, who lives next door. While the Marx family is away from the house, April sneaks onto the property, enters the enclosure where the dog is kept, plays with the dog, and leaves before the family comes home. The dog enjoyed playing with April and was in no way harmed. When the Marx family returns home, they learn these facts from another neighbor and sue April for trespass to chattels. The most likely result is judgment for: A. April, because the dog was not harmed. B. The Marx family, because April intentionally touched the dog. C. April, if it is customary in the community for young children to play with dogs. D. The Marx family, because April had been forbidden to play with the dog.

Choice A is correct

An administrative regulation of the Department of Veterans Affairs requires an appeal of a denial of benefits to be filed within 30 days. A lawyer hired to appeal a denial of benefits informally seeks to resolve the denial, but is unsuccessful. By the time it is clear that informal contacts will not resolve the problem, more than 30 days have passed. Because the deadline has elapsed, the denial of benefits is final and unappealable. Assume that in a malpractice action by the aggrieved client against the lawyer, the evidence will conclusively show that if the appeal had been timely filed, it would have been successful. In the malpractice action, which of the following is most likely: A. The violation of the administrative regulation is irrelevant if the court determines that the provision was intended only to improve administrative efficiency and to reduce the heavy workload of the department. B. The violation of the administrative regulation is irrelevant if it does not expressly create a civil cause of action. C. The violation of the administrative regulation establishes that the lawyer committed malpractice. D. The violation of the administrative regulation establishes strict liability.

Choice A is correct.

Angela is charged with cheating on an exam. She attends a meeting of the Honor Court to present her evidence and clear her name. The hearing takes place in a windowless room and lasts six hours. At the end, the Honor Court finds Angela "not guilty." Angela leaves the room and then files suit for false imprisonment against Bepo, the student who made the complaint about cheating that led to the Honor Court proceedings. A court will most likely enter judgment on the false imprisonment claim for: A. Bepo, because Angela voluntarily participated in the Honor Court. B. Angela, if she felt morally coerced to participate in the hearing in order to clear her name. C. Angela, if the evidence shows that Bepo filed the complaint because he envied the good grades Angela was earning in school. D. Bepo, if he made a mistake about the identity of the person whom he saw cheating on the exam.

Choice A is correct.

Davon is one of six companies that independently manufacture the drug Pristin, which is used to combat symptoms of the flu. The drug is sold under various names, and the pills would be viewed by consumers as similar, but not identical, in terms of their appearance. After taking Pristin, Jian becomes severely ill because the drug was contaminated during the manufacturing process with an ingredient to which some persons, like Jian, are allergic. If Jian sues Davon for negligence, the court will rule: A. For Davon, if a manufacturer other than Davon produced the dosages of Pristin that were taken by Jian. B. Against Davon, if one of the six companies produced the dosages of Pristin that were taken by Jian. C. Against Davon, if the dosage of Pristin consumed by Jian was unreasonably dangerous to a user or consumer. D. Against Davon, if the FDA withdrew its approval of Pristin before Jian took that dosage of Pristin that made him ill.

Choice A is correct.

Doris, a cleaning lady, steals a vase from the home of Jose, then sells the vase to Raphael, who pays a fair amount for the vase and is unaware that it was stolen. If Jose sues Raphael for conversion of the vase, the most likely result is judgment for: A. Jose, because his rights are superior to those of Raphael. B. Raphael, because he had no intent to interfere with the rights of Jose. C. Raphael, because his bona fide purchase of the vase cut off Jose's rights. D. Raphael, because he in no way aided, abetted, or ratified Doris's wrongful conduct.

Choice A is correct.

Harry invites Carlos to his house to help remove a large tree. Carlos, while using his chain saw, fails to exercise reasonable care. The tree falls in the wrong direction, crossing the property line and striking the neighboring house. If the neighbor sues Carlos for damages in a jurisdiction adhering to the common-law premises-liability categories, the most likely result is judgment for: (A) The neighbor, because Carlos failed to exercise reasonable care. (B) Carlos, because he enjoys the same rights as Harry under the law of premises liability. (C) The neighbor, because a possessor is liable for all harm caused to persons outside the premises. (D) Carlos, because the underlying conduct took place on Harry's land where Carlos was rightfully present.

Choice A is correct.

Homer, who recently purchased a tract of land, walks around the edge of the property. He relies upon the boundary markers set by the surveyor. Because the surveyor erred in marking the boundaries, Homer was actually walking on the land of Felix. Felix sues Homer for trespass to land. A court will most likely enter judgment for: A. Felix, regardless of whether Homer damaged Felix's land. B. Felix, but only if Felix can prove that the land was damaged. C. Homer, because he did not intend to trespass. D. Homer, because it was not unreasonable to rely upon the boundary markers set by the surveyor.

Choice A is correct.

Michael, age 13, in an act of vandalism, throws a stone at Ruth's new car and chips the paint. Assume that the law of the jurisdiction includes a statute that is identical to the following provision from the Texas Family Code: §41.001 Liability A parent ... of a child is liable for any property damage proximately caused by ... the wilful and malicious conduct of a child who is at least 12 years of age but under 18 years of age. §41.002 Limits of Recovery Recovery for damage caused by wilful and malicious conduct is limited to actual damages, not to exceed $15,000 per act, plus court costs and reasonable attorneys' fees. If Michael is sued for trespass de bonis asportatis will Michael's parents be held liable for compensatory damages caused by Michael? A. Yes, because the statute imposes vicarious liability. B. Yes, because even though the statute does not apply, parents are liable under the common law when a minor child commits any of the five torts descended from the writ of trespass. C. No, unless there is evidence that the parents failed to exercise reasonable care to prevent Michael from causing harm of this type. D. No, because the evidence is insufficient to prove trespass de bonis asportatis.

Choice A is correct.

A careless staff member of the university accidentally locked a student in the library at the end of the day. The student was not physically harmed. The student discovered that the main entrance to the building was locked and remained in the building over night. The student did not know that another exit from the building was unlocked. However, if the student had conducted a reasonable search of the building, the student would have found a door with a release bar and could easily have exited the building. The jurisdiction has adopted modified comparative fault. If the student sues the staff member for false imprisonment, the most likely result is judgment for: A. the staff member, because under respondeat superior the university, not the staff member, is responsible for any torts committed by the staff member. B. the staff member, because the staff member lacked the necessary intent. C. the staff member, because the action is barred by comparative fault. D. the staff member, because the student was not harmed.

Choice B is correct.

Opposing counsel has proffered the following instructions in a negligence action: (1) To recover for negligence, a plaintiff must prove that the defendant breached a duty owed to the plaintiff, and that the breach caused damage. Where no actual loss has been incurred by the plaintiff, the law presumes nominal damages, and a token amount may be awarded to vindicate the plaintiff's technical rights. (2) In determining whether the defendant's conduct was reasonable, you may consider the likelihood that the risk would come to fruition. However, you may not find the defendant liable unless the damage, viewed prospectively, was more likely than not to occur. (3) There can be no liability for negligence if the defendant acted in good faith to the best of his or her judgment. (4) A child of tender years, who is not engaged in an activity that is common only to adults, inherently dangerous, or requiring adult qualifications, is held to the standard of care of a child of like age, intelligence, and experience. If you object to all four submissions as misstatements of the law, a trial judge will most likely sustain (agree with) your objection to: A. 1, 2, and 3 only B. 2 and 3 only C. 1, 2, 3, and 4 D. 1 and 3 only E. 1, 2, and 4 only

Choice A is correct.

While shopping for groceries, Mildred slipped on a grape, fell to the floor, and sustained injuries. If Mildred sues the grocery store for negligence, the least relevant fact will be that: A. Mildred was in the process of petty shoplifting at the moment she slipped on the grape. B. A teenager employed by the store in violation of the child labor law saw the grape on the floor several minutes before Mildred slipped on it. C. Similar accidents have frequently occurred in the same part of the defendant grocery store. D. There was no walk-off safety mat in the fruit department near the grape bin, where the accident occurred, contrary to a state safety statute.

Choice A is correct.

As Garbo was entering her place of work, Rolf called her on her cell phone. When she answered, he told her to turn around. She did so and, when she looked across the street, she saw Rolf standing 200 feet (60 meters) away on a balcony waiving a pistol. At that very moment, over the cell phone, Rolf told Garbo that unless she agreed to marry him, he would kill either himself or her, although he was not sure which one. Garbo was afraid that he might carry out his threat, but when she walked away nothing happened. If Garbo sues Rolf for assault, she will: A. Not prevail, if she believed that Rolf was too far away to shoot her at the time she made the threat. B. Not prevail, if in fact Rolf's gun was unloaded. C. Not prevail, because the threat was conditional and uncertain. D. Not prevail, because nothing happened when she walked away. E. Not prevail, if she was unsure that Rolf would carry though his threat.

Choice A is correct. If Garbo in fact believed that Rolf was too far away to do harm, she did not suffer apprehension of imminent physical contact.

Evelyn had finished shopping at Wal-Mart and was ready to leave the parking lot. She got into her car, started the ignition, put the car into reverse, and pushed the accelerator lightly. The car started to move backward but almost instantaneously rolled over a homeless man who was sleeping on the pavement directly behind the car. Evelyn, who had approached the car from the front, rather than from the rear, did not see the man before she started to move the car. If the man survives and sues Evelyn for battery, the most likely result is judgment in favor of: A. Evelyn, because she did not see the man. B. The man, because Evelyn intentionally moved the car in reverse. C. The man, because Evelyn was mistaken about whether anything or anyone was behind the car. D. The man, because once the car was placed into reverse it was substantially certain to strike the man.

Choice A is correct. There is no evidence of intent to cause contact or of wrongful conduct that could support a transferred intent argument.

Sachel owns a building. He previously used it for his residence for twenty years. Since moving to England three years ago, Sachel has rented the building for use as a shop. The turnover rate has been high. Sachel has had three different tenants. The first two, Aaron and Beto, left at the conclusion of their successive one-year leases, and the third, Carmen, is thinking about leaving when her one-year lease expires. For all three tenants, Sachel used the same written lease form. The document grants the tenant the right to exclusive possession and control of the property and requires the tenant to keep the property in good repair. During the last three years Sachel has not visited the property, has not been asked to make any repairs, and has not been told that any repairs have been made. Palmer, a pedestrian walking outside the building, was struck and injured when an awning hanging over the entrance pulled loose from the face of the building and fell to the ground. After the collapse of the awning, Carmen had all of the debris taken to the city dump where it was irretrievably discarded. As a result, Palmer's lawyer was unable to have the awning apparatus inspected to determine whether the awning fell as a result of manufacturing defects, defective repairs, or improper installation. Assume that Palmer sues Carmen for negligent spoliation of evidence in a jurisdiction which has recognized that cause of action. Which of the following factors, if supported by the evidence, will be least relevant to (i.e., least useful in) determining whether Carmen can be held liable: A. Carmen did not intend to interfere with Palmer's case and in fact never thought about whether the wrecked awning would be important to litigation. B. It was foreseeable at the time that the awning was discarded that Palmer might bring a personal injury action to recover for his injuries. C. The evidence that was discarded was the only evidence of its kind. D. Carmen needed to remove the wreckage of the awning from the sidewalk to prevent harm to other pedestrians, and because the wreckage was heavy, bulky, and tangled, it could not have been easily stored.

Choice A is correct. Those facts relate to intentional, rather than negligent, spoliation.

Luke receives a call from Sako, a member of a gang, who tells him that the gang has taken his wife hostage. The gang demands $500,000. Luke says he does not have $500,000, but gives the gang a painting by Frieda Kahlo in exchange for his wife's release. However, when the hostage is set free, Luke learns that the woman who was being held hostage was not his wife, but another woman who was married to Lyman. The gang had called the wrong husband, and had really intended to extort money from Lyman, not Luke. The gang never discovered the mistake until after the painting had been received and the hostage set free. The painting is worth considerably more than $500,000. If Luke sues Sako for replevin, the most likely result is judgment for: A. Sako, because Luke consented to the transfer of the painting. B. Luke, if Sako has possession of the painting. C. Sako, because the gang intended to extort money from Lyman, not Luke. D. Luke, in an amount equal to the market value of the painting, regardless of whether the art work is in Sako's possession.

Choice B is correct

A motorist involved in an auto accident was using a cell phone at the time of the accident. If the plaintiff pedestrian argues that using a cell phone while driving is negligent, the plaintiff must show: A. that using a cell phone while driving is prohibited by statute. B. that using the cell phone multiplied the chances that the driver would cause an accident. C. that using a cell phone made it more likely than not that an accident would occur. D. the ordinary persons do not use cell phones while driving.

Choice B is correct. See Reynolds v. Texas.

A disgruntled law student, Cafo, who hates his Legal Ethics professor and some, but not all, of his classmates, decided to plant a bomb in the classroom. Cafo built a bomb, but was not at all sure that it would work. Whether the bomb really worked didn't matter too much, because Cafo mainly wanted to scare the professor and the students whom he disliked. Cafo planted the bomb in the classroom. Cafo also called the Dean's office with a "bomb scare," assuming that the message would be relayed and that the classroom would have to be evacuated. However, the Dean's office never relayed the threat, and the bomb did not explode at the expected time. A day later, the bomb was found by a cleaning person, who called the police to dispose of it. The police said that except for a bad timer, the bomb could have caused great harm. When the professor and students subsequently learned about the bomb that might have exploded, they were angered and distressed. If Dolf, a student in the class whom Cafo actually liked and did not want to harm, sues Cafo for assault, the most likely result is judgment for: A. Cafo, because he did not intend to harm Dolf. B. Cafo, if Dolf did not learn of the bomb until after it had been removed and disarmed. C. Cafo, because he was not sure that the bomb would explode. D. Cafo, because the bomb did not cause physical harm to Dolf.

Choice B is correct.

A state statute makes it a criminal offense for an adult to induce a minor to participate in a public or recorded sexual performance. The minor's participation in the production is not itself criminal. The statute provides: Subdivision 2. Cause of action. A cause of action exists for injury caused to a minor in a public or recorded sexual performance. The cause of action on behalf of the minor exists against a person who induces the minor to engage in the public or recorded sexual performance. Soldo, an adult, induces Rappy, a minor, to participate in the making of a pornographic film. During the filming, a ladder falls and strikes Rappy, causing injuries. There is no criminal prosecution based on these events. If Rappy brings a tort action for damages against Soldo based on the statute to recover for his injuries, the most likely result is judgment for: A. Soldo, because a civil action may not be predicated on a criminal statute unless the defendant first has been criminally prosecuted and found guilty of violating the statute. B. Rappy, if the statute was intended to prevent this type of harm. C. Soldo, because Rappy was a willing participant in the making of the film. D. Rappy, because contributory negligence is not a defense to an action based on violation of statute.

Choice B is correct.

A state statute provides: § 11-3.2(a)(1) (1) No cause of action for injury to person or property is lost because of the death of the person liable for the injury. For any injury, the action may be brought or continued against the personal representative of the decedent, but punitive damages shall not be awarded nor penalties adjudged in any such action brought to recover damages for personal injury.... (2) .... (b) .... No cause of action for injury to person or property is lost because of the death of the person in whose favor the cause of action existed. For any injury an action may be brought or continued by the personal representative of the decedent.... § 11-3.3 Limitations upon recovery where injury causes death (a) Where an injury causes the death of a person the damages recoverable for such injury are limited to those accruing before death and shall not include damages for or by reason of death, except that the reasonable funeral expenses of the decedent, paid by the estate or for the payment of which the estate is responsible, shall be recoverable in such action. The damages recovered become part of the estate of the deceased. P and D, traveling in separate cars, were involved in a fatal auto accident, and both died within two hours. P's estate sues D's estate. Under the statute, even if D is held to be responsible for the accident, it is unlikely that in a survival action P's estate can recover: A. Damages for destruction of P's car. B. P's lost future earnings. C. Damages for P's conscious pain and suffering between the impact and P's death. D. Damages for the medical expenses incurred for P's treatment prior to death.

Choice B is correct.

A woman attended a holiday party, at which she consumed several glasses of wine. After leaving the party in the car she was driving, the woman was involved in an auto accident. As a result of the injuries she sustained, the woman, who was pregnant, suffered the miscarriage of a fetus that had reached the point of viability. A wrongful death action was then filed by the woman against the party host in a state that has adopted social host liability for injuries to guests and third persons caused by alcohol served to a guest. Which of the following statements is most accurate: (A) The host cannot be held liable unless the drinks were sold to the woman. (B) The host cannot be held liable if the host reasonably believed that the woman had capacity to operate her car carefully. (C) The host cannot be held liable because the child was not born alive. (D) The host is liable because the fetus had reached the point of viability.

Choice B is correct.

ABC Armour Co. manufactures bullet-proof vests for use by police and security personnel. ABC offers several models, some providing front and back protection only, and others providing wrap-around protection. State A highway patrol officials chose to purchase the model that provides front and back protection only. Robert, a state trooper, was shot and killed while making a routine traffic arrest. The bullet entered the side of his body, where the vest did not provide protection. In a subsequent suit governed by the law as set forth in the Restatement, Third, of Torts: Products Liability, Robert's legal representative argues that the design of the vest is defective because it does not provide wrap-around coverage. Which of the following statements is most correct: (A) Proof of the availability of a wrap-around vest, by itself, provides an adequate legal basis for the jury to conclude that the vest offering only front-and-back protection was defective. (B) Proof that an available wrap-around vest is less expensive, allows greater flexibility of movement, and is more comfortable and durable than a vest with only front-and-back protection would provide an adequate legal basis for the jury to conclude that the vest offering only front-and-back protection was defective. (C) The vest offering only front-and-back protection cannot be found to be defective unless it was more dangerous than a consumer would expect. (D) The manufacturer owed no duty of care to Robert if he did not purchase the vest and his identity was unknown to the manufacturer.

Choice B is correct.

After class, Roberto walks up to John and says "If you ask one more stupid question in Torts class, I am going to grab you by the throat and break your neck." Can John recover from Roberto for assault? A. Yes, if John believes that Roberto will use force to carry out the threat. B. No, if the class will not meet again for two more days. C. No, if John believes that Roberto is physically incapable of causing serious harm. D. Yes, if Roberto is physically capable of harming John.

Choice B is correct. The threat is conditional. There is no risk of imminent physical contact.

Acting in good faith, P borrowed D's laptop, after D agreed to lend it to P for two weeks. At the end of the two weeks, P refused to return the laptop because he had not yet finished writing the research paper he was working on. However, D found the laptop at an unattended carrel in the law library. D removed the floppy diskette labeled "P's research paper" that P had inserted into the laptop's floppy disk drive and threw the diskette into the trash. The diskette was never seen again. D then grabbed the laptop and took it home with him. If P sues D for damages caused by the loss of the floppy diskette, the court is likely to enter judgment for: A. D, because D had a privilege to recapture chattels. B. P, because D intentionally discarded the floppy disk. C. D, because P refused to return the laptop when requested to do so. D. P, because it was necessary for P to use the laptop to finish the research paper and no harm was caused to the laptop.

Choice B is correct.

Alice, a stock broker, invited Balthasar, one of her clients, to hunt at her ranch in the Hill Country on the first day of hunting season. She told Balthasar that the visit to the ranch would be a "working holiday" and that she intended to discuss his portfolio after they finished hunting. Later, Cecilia, a neighbor of Alice, asked Alice whether she could hunt on the ranch on the first day of hunting season. Reluctantly, Alice agreed. No one expected Cecilia to pay for the privilege of using the land. Alice was just doing a "favor" for a neighbor. Alice did not tell Balthasar that Cecilia or anyone else would be hunting on the land on the first day of hunting season, nor did Alice tell Cecilia that Balthasar or anyone else would be hunting. While hunting, Balthasar and Cecilia both saw a deer in a clearing on Alice's land. Balthasar was in the woods west of the deer and Cecilia was in other woods east of the deer. Cecilia and Balthasar both fired simultaneously, and both missed the deer. However, Balthasar's gun shot struck Cecilia, and Cecilia's gun shot struck Balthasar. Balthasar and Cecilia have each sued Alice for negligence. The jurisdiction adheres to the common law categories with respect to premises liability. A statute in the jurisdiction that is identical to the California Recreational Use statute provides: An owner of any estate or any other interest in real property, whether possessory or nonpossessory, owes no duty of care to keep the premises safe for entry or use by others for any recreational purpose or to give any warning of hazardous conditions, uses of, structures, or activities on such premises to persons entering for such purpose, except as provided in this section. A "recreational purpose," as used in this section, includes such activities as fishing, hunting, camping, water sports, hiking, spelunking, sport parachuting, riding, including animal riding, snowmobiling, and all other types of vehicular riding, rock collecting, sightseeing, picnicking, nature study, nature contacting, recreational gardening, gleaning, hang gliding, winter sports, and viewing or enjoying historical, archaeological, scenic, natural, or scientific sites. An owner of any estate or any other interest in real property, whether possessory or nonpossessory, who gives permission to another for entry or use for the above purpose upon the premises does not thereby (a) extend any assurance that the premises are safe for such purpose, or (b) constitute the person to whom permission has been granted the legal status of an invitee or licensee to whom a duty of care is owed, or (c) assume responsibility for or incur liability for any injury to person or property caused by any act of such person to whom permission has been granted except as provided in this section. This section does not limit the liability which otherwise exists (a) for willful or malicious failure to guard or warn against a dangerous condition, use, structure or activity; or (b) for injury suffered in any case where permission to enter for the above purpose was granted for a consideration other than the consideration, if any, paid to said landowner by the state, or where consideration has been received from others for the same purpose; or (c) to any persons who are expressly invited rather than merely permitted to come upon the premises by the landowner. Nothing in this section creates a duty of care or ground of liability for injury to person or property. Will Cecilia prevail in her negligence action against Alice? (A) Yes, because she was rightfully on the property. (B) No, because she was an uninvited, non-paying guest using the land for a recreational purpose. (C) Yes, because the accident could have been prevented by the exercise of reasonable care. (D) No, because Alice is not responsible for Balthasar's actions.

Choice B is correct.

Alice, a stock broker, invited Balthasar, one of her clients, to hunt at her ranch in the Hill Country on the first day of hunting season. She told Balthasar that the visit to the ranch would be a "working holiday" and that she intended to discuss his portfolio after they finished hunting. Later, Cecilia, a neighbor of Alice, asked Alice whether she could hunt on the ranch on the first day of hunting season. Reluctantly, Alice agreed. No one expected Cecilia to pay for the privilege of using the land. Alice was just doing a "favor" for a neighbor. Alice did not tell Balthasar that Cecilia or anyone else would be hunting on the land on the first day of hunting season, nor did Alice tell Cecilia that Balthasar or anyone else would be hunting. While hunting, Balthasar and Cecilia both saw a deer in a clearing on Alice's land. Balthasar was in the woods west of the deer and Cecilia was in other woods east of the deer. Cecilia and Balthasar both fired simultaneously, and both missed the deer. However, Balthasar's gun shot struck Cecilia, and Cecilia's gun shot struck Balthasar. Balthasar and Cecilia have each sued Alice for negligence. The jurisdiction adheres to the common law categories with respect to premises liability. A statute in the jurisdiction that is identical to the California Recreational Use statute provides: An owner of any estate or any other interest in real property, whether possessory or nonpossessory, owes no duty of care to keep the premises safe for entry or use by others for any recreational purpose or to give any warning of hazardous conditions, uses of, structures, or activities on such premises to persons entering for such purpose, except as provided in this section. A "recreational purpose," as used in this section, includes such activities as fishing, hunting, camping, water sports, hiking, spelunking, sport parachuting, riding, including animal riding, snowmobiling, and all other types of vehicular riding, rock collecting, sightseeing, picnicking, nature study, nature contacting, recreational gardening, gleaning, hang gliding, winter sports, and viewing or enjoying historical, archaeological, scenic, natural, or scientific sites. An owner of any estate or any other interest in real property, whether possessory or nonpossessory, who gives permission to another for entry or use for the above purpose upon the premises does not thereby (a) extend any assurance that the premises are safe for such purpose, or (b) constitute the person to whom permission has been granted the legal status of an invitee or licensee to whom a duty of care is owed, or (c) assume responsibility for or incur liability for any injury to person or property caused by any act of such person to whom permission has been granted except as provided in this section. This section does not limit the liability which otherwise exists (a) for willful or malicious failure to guard or warn against a dangerous condition, use, structure or activity; or (b) for injury suffered in any case where permission to enter for the above purpose was granted for a consideration other than the consideration, if any, paid to said landowner by the state, or where consideration has been received from others for the same purpose; or (c) to any persons who are expressly invited rather than merely permitted to come upon the premises by the landowner. Nothing in this section creates a duty of care or ground of liability for injury to person or property. Will Balthasar prevail in his negligence action against Alice? (A) No, because he was a non-paying invitee. (B) Yes, if the jury finds that Alice knowingly failed to disclose that another person was hunting on the property the same day, and that the non-disclosure was unreasonable and a legal cause in the harm Balthasar suffered. (C) No, because a guest must take the premises "as is" and may not expect special precautions to be taken to ensure the guest's safety. (D) Yes, because Alice is responsible for Cecilia's actions.

Choice B is correct.

Alistair asks the librarian at the law library if he can leave his laptop at the front desk until he returns from dinner. The librarian agrees. Later, another student, Plato, who over heard the conversation between Alistair and the librarian, tells the librarian that Alistair sent him to pick up the laptop. The librarian turns the laptop over to Plato, who in fact had no authority to retrieve it. Plato and the laptop are never seen again. If Alistair sues the law library for conversion, the most likely result is judgment for: A. The library, because it is not liable for the conduct of the librarian. B. Alistair, because the librarian delivered the laptop to the wrong person. C. The library, if the librarian's mistake about whether Plato had permission to retrieve the laptop was reasonable and made in good faith. D. The library, because it no longer has possession of the chattel

Choice B is correct.

In a state that adheres to the public-duty rule, a medical examiner carelessly fails to correct an erroneous autopsy report about a child. As a result, a homicide investigation is commenced against the child's father, who was not responsible for the death of the child. After the investigation is terminated without prosecution, the father sues the city for negligence based on the examiner's conduct, seeking damages for mental distress, lost wages, and certain related pecuniary losses. If the city argues that the suit should be dismissed because there was no "special relationship" with the father, which of the following factors, if supported by the evidence, will be least relevant to a decision on whether or not the motion should be granted: (a) The medical examiner's actions did not induce any reliance by the father. (b) The father was a taxpayer. (c) The examiner did not know that the father or anyone else had become a suspect in case. (d) No statute imposes a duty on medical examiners for the benefit of a limited class of persons of which the father is a member.

Choice B is correct.

In which case is the defendant least likely to be held liable: A. The defendant uses greater force than is actually or apparently necessary to defend himself from an attack, causing unnecessary injuries to the plaintiff. B. The defendant in good faith, but erroneously, misidentifies a criminal suspect, with the result that the plaintiff, who is innocent of the crime, is held in police custody for 17 days. C. The defendant, whose backpack is stolen, immediately chases after the thief, but erroneously tackles the plaintiff, who is the thief's identical twin sister who played no role in the theft. D. The defendant, while being chased by a dangerous dog, enters the plaintiff's land without permission and tramples the plaintiff's flowers while attempting to escape from the dog.

Choice B is correct.

In which following cases is the defendant least likely to be judged according to an ordinary-reasonable-prudent-adult standard of care: A. A child eight years of age accidentally shoots a farmer while hunting on the first day of deer season. B. A child six years of age injures an elderly woman while skate-boarding on a parking lot regularly used as an after school playground. C. An accident prone man with a slightly lower than average IQ knocks a vase from a second story window and injures a pedestrian. D. An unlicenced driver with a learner's permit causes an auto accident while taking driving lessons under the supervision of a professional instructor. E. A driver confronted with an unexpected danger in the road slams on the brakes and causes a collision with the car traveling behind the driver.

Choice B is correct.

In which of the following cases is the negligence of Gerardo, a six-year-old child, most likely to be judged according to a standard of care based on a hypothetical child of "like age, intelligence, and experience": A. Gerardo is allegedly negligent in operating a power mower, the blade of which strikes and throws a rock, injuring Maria, an elderly woman. B. Gerardo is allegedly negligent while playing a sandbox and throws a handful of sand into the eyes of Maria, a five-year-old playmate. C. Gerardo is allegedly negligent in firing a shotgun while hunting in the company of his father, with the result that Maria, another hunter, is struck by shot from Gerardo's gun. D. Gerardo is allegedly negligent in building a pipe bomb, with the result that Maria, an adult not participating in the activity, is injured when the device prematurely explodes.

Choice B is correct.

Jeeves keeps a vicious pit bulldog in his backyard, which is fenced. The dog hates strangers and is likely to cause death or serious injury to anyone who enters the yard. While Jeeves, the only occupant of the house, is away, a burglar enters the backyard by crawling over the fence. The burglar is surprised by the dog, which then mauls the burglar. (In the jurisdiction, unlawful conduct on the part of the plaintiff, by itself, is not a bar to a tort action.) If the burglar sues Jeeves to recover for his injuries and proves that Jeeves intended for the dog to attack burglars, a court will most likely enter judgment for: A. Jeeves, because the burglar assumed the risk of harm by crawling over the fence. B. The burglar, because deadly force cannot be used to defend property. C. Jeeves, unless it is unlawful to have a pit bulldog in the neighborhood where Jeeves lives. D. Jeeves, because the house is used as a residence and burglars normally pose a serious risk of harm to residents.

Choice B is correct.

Jacob reasonably believed that he was under attack and that he would suffer serious bodily injury if he did not act immediately to protect himself. He drew a knife and stabbed the supposed assailant in the leg. However, the assailant was unarmed and, in fact, posed no threat of harm to Jacob. If the assailant files a battery action against Jacob, the court is likely to enter judgment for: A. The assailant, because Jacob incorrectly believed that he was under attack. B. Jacob, because he lacked intent. C. The assailant, because Jacob used excessive force. D. Jacob, because his conduct was privileged.

Choice D is correct.

Marco, a pedestrian, was injured in an auto accident caused by Angela. Prior to the accident, Angela had consumed several glasses of wine at a party. Tests following the accident established that Angel's blood-alcohol level was so high that she violated the criminal statute against driving while intoxicated (DWI). Because of the strength of the evidence against her, Angela pled guilty to the crime of driving while intoxicated and she was sentenced to a week in jail, the loss of her driver's license for 90 days, and probation for an additional year. If Marco sues Angela for negligence and attempts to introduce evidence that she violated the DWI law at the time of the accident, the most likely result is that: A. The evidence will be excluded because it is prejudicial to a fair trial of the negligence claim. B. The evidence will be admitted if the court holds that the DWI law establishes the standard of care for the negligence action because the DWI law was intended by the legislature to protect pedestrians from injuries caused by collisions involving drunk drivers. C. The evidence will be admitted only if the court finds that reasonable persons regularly obey the DWI law. D. The evidence will be excluded because Angela has already been punished for her crime and reliance on the same evidence in a civil trial would violate the constitutional guarantee of due process.

Choice B is correct.

Martha, a recent law school graduate, was injured in an auto accident that occurred after she had taken the bar examination, but before she received the news that she had passed the bar exam. At the time of the accident, Martha was employed part-time as a waitress, and she was in the process of looking for a job as an associate at a law firm. As a result of the accident, Martha suffered permanent brain injuries, which mean that she will never be employed as a lawyer or in any other line of work. If a suit on Martha's behalf proves that the defendant negligently caused the auto accident that led to Martha's injuries: A. Martha's recovery of lost future earnings is limited to the amount that a waitress would have earned during Martha's working life. B. Martha can recover lost future earnings including the amount that a lawyer would have earned during the period of time that Martha would have been employed as a lawyer had there been no accident. C. Martha cannot recover lost earnings, except for losses related to the period between the time of the accident and the time of trial. D. Martha can recover lost future earnings in the amount that a lawyer would have earned during the period between the date of the accident and the expiration of Martha's pre-accident life expectancy.

Choice B is correct.

Three bicycles, driven by A, B, and C, pass a horse simultaneously. The bicyclists are negligent because, by passing too close, they create a risk that the horse will be frightened. As a result of the commotion caused by the three bicycles, the horse throws the rider to the ground, injuring the rider. In which of the following scenarios would it be most difficult to establish that bicyclist A caused the injuries to the rider: A. Any one bicycle alone, or even two bicycles together, would not have scared the horse; three bicycles together were sufficient to scare the horse. B. One bicycle alone would not have been enough to scare the horse, but two bicycles would have been sufficient. C. Any one bicycle alone would have been enough to scare the horse. D. Prior to the events in question, A, B, and C agreed to attempt to scare the horse by riding too close.

Choice B is correct.

Tom, an amateur electrician, does occasional electrical repairs for his friends. While installing a light fixture for Sally, Tom did not have the right type of supplies with him. Rather than delay the project and obtain the proper supplies, Tom substituted other materials. Tom had done the same thing before on other projects, and there were no resulting problems. However this time, as a result of the substitution, a fire broke out a week after the completion of the work. Sally's house burned down and Sally's guest, Sam, was injured. Sam sues Tom for negligence to recover for his personal injuries. In response, Tom argues that most of the harm could have been avoided if Sam had agreed to a blood transfusion after being injured. Sam is a Jehovah's Witness whose religious beliefs preclude acceptance of a blood transfusion. At the trial of the negligence action, the judge instructs the jury first that "A person who has been injured is not permitted to recover for damages that could have been avoided through the exercise of reasonable care." Which of the following additional instructions would be most favorable to Sam (note: this question is not asking what the court held in Williams v. Bright, but simply which instruction, if given by the court, would be most favorable to Sam): A. "If you find that the plaintiff is entitled to recover in this action, then in deciding the nature and permanence of his injury and what damages he may recover for the injury, you must decide whether in refusing to have the transfusion the plaintiff acted as a reasonably prudent person would have acted under the circumstances." B. "You have to determine whether Sam acted reasonably as a Jehovah's Witness in refusing surgery which would involve blood transfusions. Was it reasonable for him, not what you would do or your friends or family. Was it reasonable for him given his beliefs, without questioning the validity or the propriety of his beliefs?" C. "In considering whether the plaintiff acted as a reasonably prudent person, you may consider the plaintiff's testimony that he is a believer in the Jehovah's Witness faith, and that as an adherent of that faith, he cannot accept any medical treatment which requires a blood transfusion. I charge you that such belief is a factor for you to consider, together with all the other evidence you have heard, in determining whether the plaintiff acted reasonably in caring for his injuries, keeping in mind, however, that the overriding test is whether the plaintiff acted as a reasonably prudent person, under all the circumstances confronting him."

Choice B is correct.

While shoplifting at Wal-Mart, Ruby slips on a small, round ball that was left in the middle of an aisle. If Ruby sues Wal-Mart for damages in a jurisdiction that adheres to the common-law categories of premises liability, the most likely result is judgment for: (A) The store, because Ruby was a trespasser. (B) Ruby, if a store detective who was watching Ruby saw that she was inattentive and likely to slip on the ball, but failed to warn her or take other action to prevent the harm. (C) The store, because a round ball on a floor is an obvious danger against which persons on the property must protect themselves. (D) Ruby, if she can prove that the ball was on the floor long enough that it should have been discovered by store employees.

Choice B is correct.

To escape a vicious dog, Jose took a short cut across Helen's property. In the process of trying to get away from the dog, Jose deliberately took the shortest path, which meant running directly through the flowers that Helen had been raising to enter in the upcoming flower contest at the county fair. The flowers were destroyed. When Helen saw that occur, she chased after Jose and hit him with a shovel. Later, Helen became severely depressed because she had worked hard all summer to raise the flowers. Helen's doctor has placed her on medication to calm her nerves, but it will probably be weeks before she is back to normal. If Helen sues Jose for trespass quare clausum fregit seeking actual damages, who will prevail? A. Jose, if he reasonably believed that the property belonged to his friend Ralph. B. Helen, if she qualifies as possessor of the property. C. Jose, because Helen had no right to resist Jose's entry. D. Helen, if comparative fault has not been adopted in the state. E. Jose, if he believed he was in danger of being bitten.

Choice B is correct. One who enters land pursuant to a privilege private necessity is liable for actual damages caused by the trepass.

A law student who is suspected for stealing a laptop computer is called to the office of the associate dean to answer questions about the matter. The student agrees to allow the associate dean to search the law student's locker. The student and the associate dean walk to the location of the locker, and the student unlocks the locker. No laptop is found in the locker, nor is there any other evidence that the student was guilty of theft. If the student sues the law school for false imprisonment relating to the period of time involving in going to the locker and search the locker, the most likely result is judgment for: A. the student, because the alleged confinement was too brief to support an action for false imprisonment. B. the law school, because the law student consented to the search. C. the student, if the student believed that cooperation with the investigation was "required," even though there was no use or threat of force or assertion of legal authority. D. the law school, because the associate dean did not intend to confine the student.

Choice B is the correct answer.

Husband dies at the hospital. At the direction of his surviving wife, a funeral home picks up the body at the hospital and transports it to the funeral home. When the wife fails to pay a previously agreed $1000 deposit for the funeral services, the funeral home takes the dead body and deposits it on the front porch of the family house, while no one is home. When wife returns to the house, she discovers husband's dead body and suffers severe emotional distress. If wife sues the funeral home for the tort of outrage, the most likely result is judgment for: A. The funeral home, because wife acted unreasonably in failing to pay the deposit. B. Wife, if a jury finds that wife suffered severe emotional distress. C. The funeral home, because wife was not present when the body was deposited on the porch. D. The funeral home, if wife suffered no physical harm as a result of the funeral home's conduct.

Choice B is the correct answer.

A local ordinance designed to promote water conservation prohibits homeowners from watering their lawns between 10:00 a.m. and 8:00 p.m. Violations are punishable by a fine up to $1000 payable to the city. The ordinance makes no mention of tort liability. Because her grass was starting to die from the summer heat and drought, Alice set her sprinkler system to water her lawn beginning at 11:00 a.m. When the system turned on at that time, one of the sprinklers started to spray water not only on the grass but also on the sidewalk. A pedestrian, Sonia, who was walking on the sidewalk, was startled by the water. While trying to get out of the way, Sonia fell and was injured. Sonia sues Alice for negligence and argues that Alice's violation of the ordinance establishes that Alice acted unreasonably. On that issue, a court is likely to enter judgment for: A. Alice, because violation of a local ordinance is irrelevant to liability for negligence, since tort liability is established on a state-wide basis rather than by local law. B. Sonia, because the violation of the ordinance directly caused her injuries . C. Alice, if the ordinance was enacted solely to promote water conservation. D. Sonia, because violation of a legislative enactment is negligence per se.

Choice C is correct.

A state statute provides: Section 1(a). No person who is related within the second degree of consanguinity or affinity to the owner or operator of a motor vehicle and who is being transported over the public highways of this State by the owner or operator of the motor vehicle as his guest without payment for such transportation, shall have a cause of action for damages against such owner or operator for injuries, death or loss, in case of accident, unless such accident shall have been intentional on the part of said owner or operator, or caused by his heedlessness or his reckless disregard of the rights of others. There shall be no such immunity for an owner or operator who is not so related to the guest. Assume that the statute is constitutional. Which of the following actions is most likely to be barred by the statute: A. The plaintiff is the brother of the defendant, who gratuitously offered to drive the plaintiff to the mall. En route, the defendant began to drag race with another car and caused an accident while traveling at a very high speed through an area with congested traffic. B. The plaintiff, an unrelated classmate of the defendant, was injured during a vacation trip in the defendant's car when the defendant carelessly drove into an unguarded excavation in the road. C. The plaintiff, who is the aunt of the defendant, was being given a free ride to the doctor's office. En route, the car's engine, which had previously caused some minor problems for the defendant, exploded and caused the car to go out of control, injuring the plaintiff. D. The plaintiff paid for the gasoline expenses on a trip to the Gulf coast in the car of her sister, the defendant. En route, the car went off the edge of the pavement and struck a guard rail, causing injuries to the plaintiff.

Choice C is correct.

A was a passenger in the airplane of B Company, a common carrier. In good flying weather the plane disappeared over the ocean, and no trace of it was ever found. There is no other evidence. Various explanations are possible, but unlikely, including mechanical failure which could not have been prevented by reasonable care, and bombs planted on the plane. In prosecuting a wrongful death action against B company, may the survivors of A rely upon res ipsa loquitur? A. No, because there is no evidence of lack of care on the part of Company B. B. No, because the evidence fails to establish to a "substantial certainty" that the tortious conduct on the part of B Company caused the crash. C. Yes, because the plane was under the control of B Company and it is reasonable for the jury to conclude that the most probable explanation is some negligence on the part of B Company. D. No, because a plaintiff may not rely on the doctrine unless he or she proves that other causes could not have been responsible for the accident.

Choice C is correct.

Alfonso is in love with Maria, who lives alone in El Paso. Maria wants nothing to do with Alfonso, and has refused to return his calls. In frustration, Alfonso leaves a vulgar message on the answering machine at Maria's home, using sexually explicit language and repeatedly threatening to inflict various sorts of horrendous physical harm on Maria that will be permanently disfiguring. The phone message is picked up not by Maria, but by her elderly mother (Mother), who Alfonso did not know was visiting from the East Coast. Upon hearing the threatening message, Mother concluded that it was a prank and did not worry. Maria soon returned home and suffered severe emotional distress when she listened to the threatening phone message. If Mother sues Alfonso for the tort of outrage, which of the following factors poses the greatest obstacle to a judgment in Mother's favor. A. The message left on the answering machine involved only threats of future harm. B. Maria was never physically injured. C. Mother thought the call was a prank and did not worry. D. Mother was not present at the time the threat was made.

Choice C is correct.

Employees of a grocery store knocked over a display of canned hams, severely denting some of the cans. The damaged merchandise was placed on sale. Four weeks later, Teresa bought one of the dented cans, prepared the ham for dinner, and served it to Patty and Kim, her guests. All three became violently ill soon thereafter and required hospitalization. Kim and Patty have sued the company that canned the ham and seek to employ a res ipsa loquitur theory in aid of proving negligence. Does res ipsa loquitur apply? A. Yes, because the harm is of a type that does not normally occur in the absence of negligence. B. No, because the defendant was not in exclusive control of the ham at the time it was served to the plaintiffs. C. No, if the facts show that the poisonous condition of the food was most likely caused by the denting of the cans by the grocery employees. D. No, if the defendant introduces credible testimony that it exercises due care in canning hams.

Choice C is correct.

George takes a shortcut across Miguel's yard after erroneously being told by Vera that Miguel would not mind. No harm is done to the land, but Miguel did indeed mind and thereafter sued George for trespass quare clausum fregit. The most likely result is judgment for: A. George, unless he was negligent in relying on Vera's assurances. B. George, because no damage was caused to the land. C. Miguel, so long as he was not responsible for the erroneous assurances given by Vera to George. D. Miguel, regardless of whether his conduct led Vera and Miguel to reasonably believe that he consented to the entry.

Choice C is correct.

In which of the following cases is the evidence least likely to support a jury finding of negligence on the part of the defendant: A. In a case involving the nighttime shooting of a convenience store patron by a criminal intruder, the plaintiff shows that the defendant store failed to have a security guard on duty even though similar stores in the community routinely have a security guard on duty at such hours. B. In a case against a homeowner involving damages to a passing car caused by the fall of a huge limb from a tree on the homeowner's property, the evidence shows that the homeowner knew that the limb had not had any leaves for many years and that the bark of the limb was discolored. C. In a case where a student slips on a fresh, yellow banana peel while in a bathroom in the law library, the evidence shows that the defendant school has a right to control the use of all parts of the building and routinely exercises such control. D. In a wrongful death action arising from the drowning of a tenant in an apartment complex swimming pool, the evidence shows that without excuse the defendant apartment complex failed to comply with a statutory obligation to have a lifeguard on duty when the pool was open.

Choice C is correct.

Jeremy's foot was severed when it was caught between the blade and the compaction chamber of a garbage truck on which he was working. The injury occurred when he lost his balance while jumping onto the back step of the garbage truck as it was moving from one stop to the next. The garbage truck, manufactured by XYZ Motor Co., has a warning in large red letters on both the left and right rear panels that reads "DANGER—DO NOT INSERT ANY OBJECT WHILE COMPACTION CHAMBER IS WORKING—KEEP HANDS AND FEET AWAY." Jeremy sues the manufacturer of the truck based on defective design in a jurisdiction adhering to the provisions of the Restatement, Third, of Torts: Products Liability. The action alleges that the truck was defective because the truck lacked a safety guard to prevent harm in cases where workers lose their balance. A court is most likely to hold that: (A) The action is barred by workers' compensation immunity because the injury was an on-the-job accident. (B) The warning was adequate to prevent harm and precludes Jeremy from seeking to establish that the truck was defectively designed. (C) Jeremy can recover for defective design if he proves that foreseeable danger could have been prevented by a feasible, affordable, alternative design of the truck. (D) Jeremy can recover even if there was no feasible alternative design for the truck because, viewed with the benefit of hindsight, the product was unreasonably dangerous.

Choice C is correct.

The most accurate statement about the relationship of custom to the issue of negligence is: A. The defendant's compliance with the safety customs in the industry is not relevant to the issue of whether defendant acted negligently on a particular occasion. B. The defendant's compliance with the safety customs in the industry bars liability for negligence. C. The defendant's departure from the safety customs in the industry normally constitutes rebuttable evidence of negligence. D. The defendant's compliance with the safety customs in the industry is an absolute bar to liability for negligence if the custom is embodied in a legislative enactment, such as a state statute or an administrative agency regulation.

Choice C is correct.

The state supreme court ruled last year that medical monitoring damages are recoverable on appropriate facts. Recently, a lawyer litigating a personal injury case based on exposure to a toxic substance failed to present a claim for medical monitoring damages. Such damages were therefore not awarded as part of the judgment, even though the suit was otherwise successful. The issue as to whether medical monitoring damages should have been awarded cannot be raised on appeal. After the conclusion of the case, the lawyer's client learned about the earlier supreme court ruling and sued the lawyer for negligently failing to request an award of medical monitoring damages. The lawyer is most likely to escape liability if the lawyer can show that: A.. The client never asked the lawyer to seek an award of medical monitoring damages in the toxic tort action while the suit was being litigated. B. Most lawyers in the state are unaware of the supreme court ruling that now permits an award of medical monitoring damages. C. An ordinary member of the legal profession in good standing would not have presented a claim for medical monitoring damages in a case involving these facts. D. Most states do not permit an award of medical monitoring damages.

Choice C is correct.

Three hunters, A, B, and C, each negligently shoot the victim while hunting. The wounds are inflicted simultaneously. As a consequence of the three bullet wounds, each of which is equally injurious, the plaintiff dies. In which of the following scenarios would it be most difficult to establish that hunter A caused the death of the victim: A. One bullet wound alone, or even two bullet wounds together, would not have caused the death of the victim; but three bullet wounds together were sufficient to cause the death B. Any one bullet wound alone would have been sufficient to cause the death. C. One bullet alone wound would not have been enough to cause the death, but two bullet wounds together would have been sufficient. D. Prior to the events in question, A, B, and C agreed to attempt to scare the victim by firing in the victim's direction.

Choice C is correct.

Tom, an amateur electrician, does occasional electrical repairs for his friends. While installing a light fixture for Sally, Tom did not have the right type of supplies with him. Rather than delay the project and obtain the proper supplies, Tom substituted other materials. Tom had done the same thing before on other projects, and there were no resulting problems. However this time, as a result of the substitution, a fire broke out a week after the completion of the work. Sally's house burned down and Sally's guest, Sam, was injured. Assume that Sally sues Tom for negligence seeking damages for the destruction of her house. Tom cannot be held liable for negligence: A. If Tom believed honestly and in good faith that the substitution of materials would be effective and that no harm would result. B. If the substitution of materials was less likely than not to cause a fire. C. If the substitution was reasonable and did not pose a foreseeable risk of harm. D. Because he is not a professional electrician.

Choice C is correct.

Which of the following statements is most correct in states that adhere to the common-law categories of trespasser, licensee, and invitee: (a) Possessors of land always owe minor children a duty of reasonable care. (b) Possessors of land never owe an undiscovered trespassing minor child a duty of reasonable care. (c) A possessor of land must use reasonable care to protect a trespassing minor child from highly dangerous artificial conditions. (d) A possessor of land must exercise reasonable care to protect a trespassing minor child only from dangerous conditions that are likely to attract the child onto the property.

Choice C is correct.

Which of the statements is most accurate with respect to spoliation of evidence: A. A party (unlike a third party) cannot be sanctioned (penalized) for spoliation of evidence because litigation is "adversarial" and a party has no duty to protect the interests of the opposing party. B. Spoliation of evidence by a party to litigation may be sanctioned (penalized) by a court only if the party's conduct violates applicable criminal law provisions relating to destruction of, of tampering with, evidence. C. A court may dismiss a lawsuit brought by party who is found to have destroyed evidence critical to evaluation of the merits of the suit. D. A court may not impose sanctions based on spoliation of evidence that occurred prior to the commencement of a lawsuit. E. Sanctions for spoliation of evidence are imposed without regard to whether the spoliation was culpable or innocent.

Choice C is correct.

Wife was driving her car in which husband was a passenger. Another car loaded with teenagers, driven by the defendant, ran a red light and hit wife's car. Wife was seriously injured, but husband was not injured. Assume that a jury will find that wife was 5% responsible for the accident, defendant was 95% responsible, and husband was in no way at fault. If husband sues defendant for the value of the lost sexual companionship caused to him by reason of the physical injuries to his wife, a jury is most likely to enter judgment for: A. Husband, for 100% of the damages. B. Defendant, because such damages are not recoverable in a tort action. C. Husband, for 95% of the damages. D. Defendant, because husband was not injured and therefore such damages are not recoverable.

Choice C is correct.

A young lawyer works for a law firm. In the course of dealing with one of the firm's clients, the lawyer persuades the client to have sex with the lawyer in exchange for not being billed by the law firm for certain work performed on the client's case. The law firm is unaware of the sex-for-services arrangement between the young lawyer and client, and nothing like that was foreseeable. If the client regrets the sexual relationship and sues the law firm for the tort of outrage, the most likely result is judgment for: A. the law firm, because the conduct of the young lawyer was intentional, rather than negligent. B. the client, because the conduct of the young lawyer was extreme and outrageous. C. the law firm, because the arrangement was unknown to the law firm and not intended to further the business purposes of the law firm. D. the client, because the law firm is strictly liable for the conduct of its employees.

Choice C is correct. See Western Union v. Hill.

A and B are independent companies that install swimming pools. If the bedrock is too close to the surface of the ground, they excavate by using dynamite to blast out the rock. Assume that while working in the plaintiff's neighborhood, A and B independently set off explosions simultaneously and that each is negligent by reason of using explosives that are too strong for the job. As a result of the twin explosions, the plaster in plaintiff's house down the street is severely cracked. If the plaintiff sues A and B to recover the cost of having the house re-plastered, which of the following statements is least likely to be deemed correct with respect to factual causation: A. A and B are each a factual cause of the damages if the negligent conduct of either one alone would have cracked the plaster. B. A and B are each a factual cause of the damages if the negligent conduct of either one alone would not have been sufficient to crack the plaster, but the conduct of both together was sufficient to crack the plaster. C. Without evidence as to whether the negligent conduct of A or the negligent conduct of B made the greater contribution to the cracking of the plaster, it is impossible to tell whether A or B was a factual cause of the harm. D. The negligence of A or B cannot be found to be a factual cause of harm to the plaintiff unless that negligence substantially multiplied the chances of cracking the plaintiff's plaster.

Choice C is the best choice since the other statements are correct.

A housekeeper unplugs the air pump for an aquarium while vacuuming the carpeting, but forgets to plug it back in. As a result, the plaintiff homeowner's prized fish die from lack of oxygen. If the plaintiff sues the housekeep to recover for the loss, which of the following statements is least correct: A. An action for conversion will fail. B. An action for negligence will permit recovery of the market value of the fish. C. Damages in a wrongful death suit will be limited to pecuniary losses. D. Punitive damages cannot be recovered.

Choice C is the best choice. A wrongful death action will not lie for the death of a pet.

Garbo was constantly being hounded by her former boyfriend, Rolf, whom she hoped she would never see again. Rolf has persisted in calling Garbo's apartment at all hours of the night, following Garbo in public, asking her friends for information about her, and sending her unwanted love letters in the mail. Garbo was so distraught over Rolf's conduct that she was afraid to leave her apartment for fear that she would meet him on the street and that he would insist on asserting his love for her. Unknown to Rolf, Garbo stayed in her apartment for five days. Assume that Garbo sues Rolf for false imprisonment relating to the five days she stayed in the apartment. Which of thee A. Garrbo will lose if she voluntarily chose to remain in her apartment to avoid meeting Rolf on the street. B. Garbo will lose if Rolf never used or threatened to use force, or asserted any form of legal authority, for the purpose of confining Garbo. C. Garbo will lose unless the jury finds that Rolf's conduct was extreme and outrageous. D. Garbo will lose if she was aware that she could have left the apartment at any time without harm to her person, property, or dignity. E. Garbo will lose unless Rolf knew that Garbo would be confined in her apartment or wanted that to happen.

Choice C is the correct answer. Extreme and outrageous conduct is not an element of false imprisonment.

In which of the following cases is a tort action for damages least likely to be barred by the unlawful conduct defense: A. The plaintiff is injured while burglarizing the defendant's home in the dark of night by falling through a part of the stairs where a tread had been removed in anticipation of repairs. B. The plaintiff, who sells sex for money in a jurisdiction where prostitution is illegal, contracts an undisclosed venereal disease from the defendant customer. C. The plaintiff is shot by the defendant security guard while robbing a bank. D. The plaintiff, who was practicing law without a license, is struck and injured by a falling piece of defective plaster at the courthouse of the defendant county.

Choice D is correct

Question 1: After running laps around the track for a half hour at a gym where he had just became a member, Marco was sweaty and overheated. He headed for the locker room, where he took off his clothes and entered the shower. The shower had two unmarked chrome faucet handles, one on the left and one on the right. Marco turned the handle on the right because he wanted a to cool off immediately by taking a cold shower. To his surprise, hot water gushed from the shower head and Marco was scalded. Contrary to Marco's expectations, the handle on the right controlled the hot water and the handle on the left controlled the cold water. If Marco sues the gym for negligence, which of the following pieces of evidence will be least relevant to the issue of liability: A. Under penalty of a criminal misdemeanor fine of up to $500 per day, the state building code requires public gyms to provide showers for patrons, and further requires that if water to a shower is controlled by two handles, the handle controlling the hot water must be positioned to the left of cold water handle. B. It is customary in the community for businesses providing shower facilities for patrons to position the hot water handle on the left, if a shower is controlled by two faucet handles. C. A large sign at the entry to the shower advised users that for each shower the hot water was controlled by the handle on the right. D. The plumbing for the shower in question was installed three years earlier by a contractor over whom the gym exercised no control. E. No person had ever been scalded by the shower in question during the nearly three years that it was in use since the gym commenced operations.

Choice D is correct because it is least relevant. The circumstances relating to the initial installation are irrelevant because the gym had control of the facility at the time the accident occurred and for long enough before that date to be expected to take remedial measures.

A state statute makes it a crime to hunt animals on private property which is posted with signs saying "No Hunting." The crime is punishable by a fine not to exceed $500 for each violation. In violation of the statute, Phidias hunts on Max's land, which is posted with "No Hunting" signs. While doing so, a shot fired by Phidias's gun strikes a child playing in a forest on the property. If the child sues Phidias for the resulting injuries and produces no evidence of negligence other than the violation of the statute, the most likely result is judgment for: A. The child, if Phidias has no excuse for the violation of the statute. B. Phidias, because a criminal statute cannot form the basis for a civil cause of action and there is no other evidence of negligence. C. The child, because the statute imposes absolute liability. D. Phidias, if the court concludes that the statute was intended to prevent only harm to the property rights of owners caused by trespassing hunters.

Choice D is correct.

Bozo dares Kimbo to hit him in the abdomen as hard as he can because his abdominal muscles are rock-hard and the punch will not hurt him. Kimbo lands a forceful blow to Bozo's abdomen . Bozo doubles over, falls to the ground, and is rushed to the hospital with internal bleeding. As a result of his injuries, Bozo misses two weeks of work and incurs $3000 in medical expenses. If Bozo sues Kimbo for battery, the most likely result is judgment for: A. Bozo, because he did not consent to being injured. B. Kimbo, because recovery for the complications of the punch is barred by the doctrine of emergent circumstances. C. Bozo, because the eggshell-skull rule imposes liability for unforeseeable consequences of physical injury. D. Kimbo, because the action is barred by volenti non fit injuria.

Choice D is correct.

Finance Code § 304 provides: § 304.104. ... [P]rejudgment interest accrues on the amount of a judgment during the period beginning on the 180th day after the date the defendant receives written notice of a claim or on the date the suit is filed, whichever is earlier, and ending on the day preceding the date judgment is rendered. § 304.105. (a) If judgment for a claimant is less than the amount of a settlement offer of the defendant, prejudgment interest does not accrue on the amount of the judgment during the period that the offer may be accepted. (b) If judgment for a claimant is more than the amount of a settlement offer of the defendant, prejudgment interest does not accrue on the amount of the settlement offer during the period that the offer may be accepted. § 304.107. If a settlement offer does not provide for cash payment at the time of settlement, the amount of the settlement offer for the purpose of computing prejudgment interest is the cost or fair market value of the settlement offer at the time it is made. On Day 1, plaintiff is negligently burned by the defendant. On Day 20, plaintiff sues the defendant to recover the cost of medical care and serves the complaint on the plaintiff's attorney. Prior to filing suit, plaintiff made no claim to defendant seeking payment. By day 90, plaintiff's injuries are healed, plaintiff has returned to good health, medical care is complete, and no further medical attention is required. On Day 100, the defendant makes a written offer to pay the plaintiff $50,000. Plaintiff never accepts the offer, which remains open until trial. On Day 201, the jury awards plaintiff $10,000 in damages. With respect to prejudgment interest, plaintiff can recover: A. 80 days of interest on $50,000. B. Nothing. C. 180 days of interest on $10,000. D. 80 days of interest on $10,000.

Choice D is correct.

Lucinda, an employee at the drug store, was accused by her boss of stealing company property and threatened that if she was guilty she would lose her job. She was never threatened with physical harm. For the purpose of clearing her name and to prevent being fired, Lucinda agreed to be interrogated by the personnel supervisor and authorized a search of her purse. The two-hour interrogation and search turned up no evidence of theft of company property and Lucinda was permitted to continue her job. However, Lucinda decided she could no longer work at the drug store. She quit her job and then sued the drug store for false imprisonment relating to her participation in the investigation. The most likely result is judgment for: A. Lucinda, if she believed she would be fired if she did not agree to participate in the investigation. B. The drug store, because the alleged tort occurred at the workplace and therefore an action is barred by worker's compensation. C. Lucinda, because she was not guilty of theft. D. The drug store, because Lucinda voluntarily participated in the investigation that caused the alleged detention

Choice D is correct.

Martha lent Wendy a copy of her Torts I outline after Wendy promised not to let anyone else have a copy. Wendy left the outline unattended at her research carrel. Jeffrey found it and made copies of the outline, which he then distributed to everyone in the class. If Martha sues Wendy for conversion, a court will most likely enter judgment for: A. Martha, because Wendy's conduct resulted in a serious invasion with Martha's rights. B. Wendy, because the harm that resulted was concerned with intangible intellectual property rights (the content of the outline), rather than a chattel belonging to the plaintiff. C. Wendy, because the possibility that someone else would obtain a copy of the outline was an inherent risk that Martha assumed by lending the outline to Wendy. D. Wendy, because the alleged interference was a result of Wendy's negligence, rather than Wendy's unconsented intentional exercise of dominion and control over the outline.

Choice D is correct.

Martha lent Wendy a copy of her Torts I outline after Wendy promised not to let anyone else have a copy. Wendy left the outline unattended at her research carrel. Jeffrey found it and made copies of the outline, which he then distributed to everyone in the class. If Martha sues Wendy for conversion, a court will most likely enter judgment for: A. Martha, because Wendy's conduct resulted in a serious invasion with Martha's rights. B. Wendy, because the harm that resulted was concerned with intangible intellectual property rights (the content of the outline), rather than a chattel belonging to the plaintiff. C. Wendy, because the possibility that someone else would obtain a copy of the outline was an inherent risk that Martha assumed by lending the outline to Wendy. D. Wendy, because the alleged interference was a result of Wendy's negligence, rather than Wendy's unconsented intentional exercise of dominion and control over the outline.

Choice D is correct.

Quando coaches a community soccer team that consists of 12-year-old boys from the neighborhood. While working with one of the boys alone after practice, Quando took sexual liberties with the boy. The boy's parents then sued Quando in negligence to recover damages for the emotional distress they suffered upon learning that their son had been sexually abused. The jurisdiction adheres to the zone-of-danger (rather than the Dillon v. Legg/foreseeability) view. Will the parents recover? (A) Yes, because they were within the zone of danger. (B) Yes, because their son was within the zone of danger. (C) Yes, if after learning of the abuse, they feared for the emotional welfare of their child. (D) No, because they were never in physical danger.

Choice D is correct.

Under which of the following scenarios will concerted action liability be least readily established: A. A and B drag race on a public street. A's car strikes C, a pedestrian. C sues B. B. As part of a continuing course of raucous activity at a party attended only by invited guests, A and B capture C and throw him overboard from a barge, causing injuries. C sues D, the guest who suggested to A and B that they throw C into the water, shortly before they did so. C. Only three pharmaceutical companies (A, B, and C) produce a certain generic consumer drug. In marketing the drug, the companies, pursuant to a common plan, knowingly rely upon the same negligently inadequate university laboratory tests to "prove" the drug's safety. A, B, and C, in accordance with their expressly stated agreement, sell the drug in identical dosages to pharmacists within the same area of distribution. Pharmacists then use quantities of this generic drug interchangeably to fill consumer prescriptions. Ten years later, X discovers that he has been harmed by dosages of the drug purchased and consumed from a pharmacist within the mentioned distribution area. X sues A, B, and C for negligence. D. An amusement ride at a summer park unexplainably catches fire and injures children, including Z. Expert testimony establishes that the fire could have been caused only by one of two things: the negligent manufacture of an electrical component by A or the negligent manufacture of a valve by B. A and B are unrelated companies. Z sues A and B.

Choice D is correct.

Which of the following statements is true: A. Evidence of departure from custom in the community is indispensable in proving negligence. B. Conformance with custom in the community conclusively establishes that the defendant acted reasonably. C. If there is no custom in the community with respect to the events that gave rise to the plaintiff's injury, the defendant cannot be held to be negligent. D. In most states, evidence of departure from customary practice is less helpful in establishing negligence than evidence of an unexcused violation of a standard-setting statute.

Choice D is correct.

Professor Xiuwen called on a first-year law student, Yago, to walk to the front of the classroom to recite on a case. While Yago was still up front, the Professor walked to Yago's empty seat, then said to Yago, "watch this." The Professor then touched Yago's laptop computer and began to wiggle the screen back and forth in a manner that could have damaged the laptop. Fortunately, the laptop was unharmed. Yago can recover from the Professor for: I. Battery II. Assault A. I only B. II only C. I and II D. Neither I nor II.

Choice D is the correct answer.

A lawyer representing a client is entrusted with documentary evidence relating to the case. Because the documents are lost, the case cannot go forward and the client is unable to recover damages for serious injuries sustained in an industrial accident. If the client sues the lawyer for negligence, the lawyer is most likely to escape liability by persuading that jury that: A. the documents were stolen by a third party. B. the lawyer observed the same precautions that lawyers ordinarily observe with respect to the care of trial evidence. C. the plaintiff probably would have lost the personal injury case even if the documents were available and the suit had gone to trial. D. the plaintiff failed to keep copies of the documents that were entrusted to the lawyer. E. the underlying personal injury claim was barred by sovereign immunity.

Choice E is correct. If the defendant can establish that the suit was barred by sovereign immunity, nothing else matters. Choice C is tempting; but E is better because it eliminates issues relating to loss of a chance and spoliation.

A dentist performed extensive dental surgery on a patient, which was painful and expensive. Sometime after the surgery was completed and the doctor/patient relationship had ended, the patient learned that, at the time the surgery was performed, the dentist was infected with HIV. The patient, who has not yet been tested for HIV, now worries that (a) the virus was communicated to her during the surgery and (b) even if the virus was not communicated, it could have been transmitted to her and could have caused serious harm. If the patient sues the dentist for assault in a state other than Texas that has not adopted comparative negligence or comparative fault, the most likely result is judgment for: A. the dentist, because the dentist lacked intent to cause harm. B. the patient, because there is a foreseeable risk that HIV can be transmitted during surgery. C. the patient, if the jury finds that the patient's distress is reasonable and genuine. D. the dentist, because the distress did not occur until after the surgery was completed and the doctor/patient relationship ended.

D

Even if carelessness on the part of the plaintiff is established in a negligence suit in a state with modified comparative fault, the plaintiff may recover compensation for 100% of the plaintiff's injuries.

False

If an employee is intentionally battered by an employer in a jurisdiction with a worker's compensation law, the employee may not sue the employer for battery.

False

If the plaintiff is 55% percent at fault and the defendant is 45% at fault, the plaintiff will be better off in a state with a 50% (modified) comparative fault system, than in a state with a pure comparative fault system.

False

In a state with pure comparative fault, a defendant cannot be sued for an intentional tort.

False

Most tort rules are based on federal law, not state law.

False

Punitive damages may be awarded in a case based on simple failure to exercise ordinary care.

False

Statutes of limitations for intentional tort actions, such as battery, are always longer than statutes of limitations governing negligence actions

False

Strict liability may only be imposed by statute, not by a common-law decision

False

The common law doctrine of contributory negligence is now only of historical interest because it has been abandoned in all states.

False

Raymon is the sole-owner of a car-repair garage. Ivan is an electrician who works for Azar Co., which was hired by Raymon to do work at the garage. While Ivan was installing new electrical circuits in the garage, Zeto stole a car from the garage parking lot. Raymon urged Ivan to chase after the stolen car. Ivan jumped into Raymon's car, and did so. During the chase, Ivan drove negligently and caused an injury to a pedestrian. Assume that the pedestrian seeks to hold Raymon liable for the harm caused by Ivan. Which of the following statements is least correct: (A) Raymon will be liable for the harm to the pedestrian under the state's Car Owner Responsibility Law. (B) Raymon will be liable for the harm to the pedestrian if Raymon knew that Ivan had a history of reckless driving when he urged Ivan to chase the thief. (C) Raymon will be liable for the harm to the pedestrian because Ivan was an independent contractor asked to perform a task in a public place that was inherently dangerous and required special precautions. (D) Raymon will be liable for the harm to the pedestrian because Ivan was a servant of Raymon within the scope of his employment.

D is the best choice (the least correct answer).

Alice, a first-year law student, was almost late for class. Because of the construction by the classroom building, the only entrance was via a small handicapped ramp. The ramp was crowded with numerous students going in and out of the classroom building and several other students who were loitering on the ramp. To avoid being late for class, Alice abruptly pushed past the loitering students, making contact with Bello, who was offended by the rough contact that she made with him. If Bello sues Alice for battery, the most likely result is judgment for: A. Alice, if it was essential for her to push past Bello to get to class on time. B. Bello, if he was placed in apprehension of the contact before it occurred. C. Alice, if she did not injure Bello or damage his personal property. D. Bello, if Alice used unreasonable force and was certain to make contact with Bello or one of the other students using the ramp.

D is the correct answer.

Sachel owns a building. He previously used it for his residence for twenty years. Since moving to England three years ago, Sachel has rented the building for use as a shop. The turnover rate has been high. Sachel has had three different tenants. The first two, Aaron and Beto, left at the conclusion of their successive one-year leases, and the third, Carmen, is thinking about leaving when her one-year lease expires. For all three tenants, Sachel used the same written lease form. The document grants the tenant the right to exclusive possession and control of the property and requires the tenant to keep the property in good repair. During the last three years Sachel has not visited the property, has not been asked to make any repairs, and has not been told that any repairs have been made. Yesterday, Palmer, a pedestrian walking outside the building, was struck and injured when an awning hanging over the entrance pulled loose from the face of the building and fell to the ground. Palmer sues Sachel, Aaron, Beto, and Carmen for negligence. In the lawsuit based on the accident, an expert will testify that it is obvious from the debris that the apparatus attaching the awning to the building was defectively repaired on one or more occasions during the past five years, but that it is impossible to say exactly when the repairs were improperly performed. Palmer produces no evidence of any specific acts of negligence on the part of any of the defendants but seeks to hold all four jointly and severally liable for negligence based on a res ipsa loquitur theory. If prior to submission of the case to the jury Sachel moves to dismiss the action against him based on Palmer's failure to produce evidence of negligence, the court should enter judgment on the motion: A. In favor of Palmer, because all potentially responsible parties have been sued and one of them must have been negligent. B. In favor of Palmer, because awnings do not fall and strike pedestrians unless someone has been negligent. C. In favor of Palmer, because res ipsa loquitur permits an innocent plaintiff harmed by negligence to get to the jury in a suit against all potentially responsible parties. D. Against Palmer, because there is no evidence that the negligence probably occurred when Sachel used the property as his residence, and during the period of the leases Sachel had no right to possession or control.

D is the correct answer.

Today, in jurisdictions subscribing to comparative negligence or comparative fault, assumption of the risk is never treated as a limited or partial defense in actions based on recklessness, negligence, or strict liability.

False

Under Texas statutes, parents are liable for personal injuries tortiously inflicted by their children who are under the age of twelve.

False

Under common-law rules in most states, parents, by reason of parentage, are vicariously liable for the torts committed by their minor children.

False

Under the doctrine of respondeat superior, an employer is vicariously liable for all torts committed by an employee.

False

Worker's compensation statutes permit a worker injured by the negligence of a co-worker at the work site to recover an insurance award from the worker's compensation program that supplements the recovery that the worker can receive in a tort action against the employer based on respondeat superior.

False

A person acts with intent to cause a result if under the circumstances at the time of the action the result is more likely than not to occur.

False

All jurisdictions that originally adopted pure or modified comparative negligence have switched to some form of comparative fault.

False

Cohen v. Petty was the negligence case where the driver fainted after exclaiming to his wife, "Oh, Tree, I feel sick!", but was not held liable for injuries to a passenger. The case would have turned out differently, and thus the plaintiff would have recovered, if the plaintiff had argued that the doctrine of transferred intent was applicable to the case.

False

Garbo was constantly being hounded by her former boyfriend, Rolf, whom she hoped she would never see again. Rolf has persisted in calling Garbo's apartment at all hours of the night, following Garbo in public, asking her friends for information about her, and sending her unwanted love letters in the mail. As a result of Rolf's conduct, Garbo has suffered emotional distress. If Garbo sues Rolf for damages based on the tort of outrage, which of the following facts (assuming it is supported by the evidence) would be least relevant to the issue of whether the Rolf's conduct was extreme and outrageous: A. Rolf has persisted in the allegedly offensive conduct everyday for more than six months. B. Garbo has repeatedly told Rolf to leave her alone because every time she even thinks of him she becomes suicidal. C. As a result of Rolf's conduct, Garbo secretly sought medical care and has begun to take a strong anti-depressant medication. D. Rolf's following Garbo in public constitutes a violation of state criminal statute against "stalking." E. The friends who have been asked for information about Garbo report that Rolf has made threats against the personal safety of Garbo.

The answer is C. The fact that Garbo secretly sought medical care says something about the severity of her emotional distress (element 4), but says little if anything about whether the conduct (sending the love letters was "utterly intolerable in civilized society" (element 2).

A pregnant woman, Sybil, visited the apartment of her friend, Delphia, to celebrate Delphia's birthday. During the visit, Sybil was electrocuted when she flipped the light switch in the powder room, which had recently been replaced by Delphia's landlord. Delphia was unaware that the switch was dangerous, but could easily have discovered that fact if she had checked to see if the work had been properly performed by the landlord. The jolt of electrical current was so severe that Sybil went into labor. The fetus Sybil was carrying was stillborn. The fetus was at such an early stage of development that it would not have been capable of surviving outside the womb. Sybil commences a wrongful death action for negligence against the landlord based on the stillbirth of the child. The jurisdiction adheres to the traditional rules governing both prenatal injuries and the tort liability of lessors. A court will most likely enter judgment for: A. The landlord, because the landlord was not in possession of the apartment at the time the injury occurred. B. The landlord, because the unborn child was not capable of surviving outside the womb. C. Sybil, because the landlord's negligence caused the stillbirth of the child. D. The landlord, because Sybil was not present for the purpose of doing business with anyone.

The correct answer is B.

A young boy, Millo, age 11, was a member of the church choir. The director of the choir was an ordained church minister under whose direction the choir practiced. There was a close relationship of trust and confidence between Millo and the choir director, and Millo thought of him as a "second father." One day, after choir practice, Millo was helping the minister to put away the chairs. No one else was around. The minister took advantage of the occasion to sexually molest the child. As a result of the incident, Millo later suffered severe emotional distress, as did Millo's parents when they found out about the incident from the child six months later. Assume that prior to the sexual assault of Millo, the police department of the city in which the church is located had received several reports about alleged sexual abuse of choir boys by the minister but had failed to conduct any investigation. An investigation would have revealed that the minister had sexually abused several young children and would have provided the basis for criminal charges against the minister. There is no statute in the jurisdiction governing how the police are to handle such cases, but expert testimony can be adduced from police chiefs in neighboring cities that they would have promptly investigated such facts and then taken appropriate action. If the city is sued by Millo for negligent failure to act in a jurisdiction that adheres to the public-duty rule, the most likely result is judgment for: A. Millo, because the city has a duty to all members of the public, including Millo, to exercise reasonable care to prevent foreseeable harm. B. The city, because there was no special relationship between the city and Millo. C. Millo, because the testimony of the police chiefs from neighboring cities is sufficient to establish that the defendant city was negligent. D. The city, because the public-duty rule applies only to cases involving able-bodied adults.

The correct answer is B.

A doctor fails to properly test a pregnant woman. As a result, the doctor does not detect or disclose a naturally-occurring birth defect in the fetus being carried by the woman. If the birth defect had been disclosed the woman absolutely would not have had an abortion because of her religious convictions. But when the child is born with the birth defect, the woman suffers great emotional distress solely because of the impaired condition of the child. Can the woman prevail in an action against the doctor based on negligence seeking damages for emotional distress? A. No, because the doctor's negligence did not cause the woman's emotional distress. B. Yes, because the doctor was negligent and the mother's emotional distress was genuine. C. No, because the mother will retain the benefits that flow from raising the child. D. Yes, because the doctor failed to exhibit that level of care that would have been exhibited by an ordinary member of the profession under similar circumstances.

The correct answer is A.

A sport-utility vehicle with a roof rack was manufactured by a company called Maker. The SUV was then sold to a customer, named Buyer, by a car dealer, called Dealer, which sells vehicles manufactured by Maker. While the SUV was traveling down the highway part of the roof rack detached. The loose part first bounced on the roof of Buyer's SUV, denting the metal and chipping the paint. The loose part then struck a small car driven by another driver, called Other, shattering the windshield of Other's car, but not injuring the passengers. Assume for this question only that Other, while traveling behind the SUV, saw that the roof rack was beginning to detach and moved closer to the SUV to get a better look at what was happening. At that point, the roof rack detached and struck Other's car, causing property damage. Assume further that Other sues Maker in a jurisdiction adhering to pure comparative fault and proves that Maker was negligent in manufacturing the SUV and that Other was within the foreseeable zone of danger. If Maker asserts that Other's conduct constitutes some type of assumption of the risk, the most likely effect is that recovery by Other against Maker will be: A. Partially barred. B. Totally barred. C. Not affected in any way by Other's conduct. D. Totally barred if the jury finds that Other had the last clear chance to avoid the accident.

The correct answer is A.

A young boy, Millo, age 11, was a member of the church choir. The director of the choir was an ordained church minister under whose direction the choir practiced. There was a close relationship of trust and confidence between Millo and the choir director, and Millo thought of him as a "second father." One day, after choir practice, Millo was helping the minister to put away the chairs. No one else was around. The minister took advantage of the occasion to sexually molest the child. As a result of the incident, Millo later suffered severe emotional distress, as did Millo's parents when they found out about the incident from the child six months later. Assume that the minister purchased and is covered by a standard homeowner's insurance policy covering third-party liability for personal injury and property damage. A judgment in favor of Millo against the minister based on battery is most likely to be: A. Not covered by the policy, because the harm was "intended or expected from the standpoint of the insured." B. Covered by the policy, because the judgment imposes liability on the minister for injury to a third party. C. Covered by the policy, because the injuries resulted from a failure to exercise care. D. Not covered by the policy, because the injury took place away from the minister's home.

The correct answer is A.

In which of the following cases is B least likely to make out a prima facie case of premises liability against A? (In answering this question, ignore affirmative defenses and focus on the prima facie case). 1. The A Company has in its store a large front door, made of heavy plate glass. The door is well lighted and plainly visible, and its existence is obvious to any person exercising ordinary attention and perception. B, a customer in the store, while preoccupied with his own thoughts, mistakes the glass for an open doorway, and runs his head against it and is injured. 2. The A Department Store has a weighing scale protruding into one of its aisles, which is visible and quite obvious to anyone who looks. Behind and about the scale it displays goods to attract customers. B, a customer, passing through the aisle, is intent on looking at the displayed goods. B does not discover the scale, stumbles over it, and is injured. 3. A owns an office building, in which he rents an office for business purposes to C. The only approach to the office is over a slippery waxed stairway, whose condition is visible and quite obvious. B, employed by C in the office, uses the stairway on her way to work, slips on it, and is injured. 4. An incoming train of the A Railroad deposits B, a passenger, upon a platform from which the only exit is over a footbridge crossing the tracks. Employees of the Railroad have encumbered the bridge with baggage from the train. B, crossing the bridge, attempts to climb over the pile of baggage, falls, and is injured. A. Case 1. B. Case 2. C. Case 3. D. Case 4.

The correct answer is A.

A young boy, Millo, age 11, was a member of the church choir. The director of the choir was an ordained church minister under whose direction the choir practiced. There was a close relationship of trust and confidence between Millo and the choir director, and Millo thought of him as a "second father." One day, after choir practice, Millo was helping the minister to put away the chairs. No one else was around. The minister took advantage of the occasion to sexually molest the child. As a result of the incident, Millo later suffered severe emotional distress, as did Millo's parents when they found out about the incident from the child six months later. Assume that the minister had never engaged in any similar conduct prior to this incident or caused anyone to think that this could occur. If Millo seeks to hold the church vicariously liable for battery based on respondeat superior (not negligence or ostensible agency), the most likely result is judgment for: A. Millo, because the tort occurred incidental to the performance of the minister's duties as choir director. B. The Church, because the tort occurred outside the scope of the minister's employment. C. Millo, because strict liability is imposed for the purpose of deterring this type of conduct. D. The Church, because the injurious conduct was unforeseeable.

The correct answer is B.

A young boy, Millo, age 11, was a member of the church choir. The director of the choir was an ordained church minister under whose direction the choir practiced. There was a close relationship of trust and confidence between Millo and the choir director, and Millo thought of him as a "second father." One day, after choir practice, Millo was helping the minister to put away the chairs. No one else was around. The minister took advantage of the occasion to sexually molest the child. As a result of the incident, Millo later suffered severe emotional distress, as did Millo's parents when they found out about the incident from the child six months later. If Millo's parents sue the minister for negligent infliction of emotional distress they suffered in a jurisdiction that follows the rule of Dillon v. Legg, the most likely result is judgment for: A. The parents, because Millo was within the zone of danger. B. The minister, because the parents were not present and did not learn of the sexual assault until long afterward. C. The parents, because their familial relationship with Millo guarantees the genuineness of their claim. D. The minister, because the parents are secondary, rather than primary, victims of the conduct in issue.

The correct answer is B.

Cristella attended a lecture in the Alumni Room of the law school on "International Terrorism." The lecture was open to all members of the public. Cristella was accompanied by her daughter Apalla, age six. Apalla was injured while listening to the lecture when a leg on her chair unexpectedly broke and she fell to the floor. If suit is brought against the law school by Apalla in a jurisdiction which adheres to the traditional categories on premises liability, the law school is most likely to escape liability by establishing that: A. Apalla and her mother paid nothing to attend the lecture. B. The defect in the chair was unknown to the law school and could not have been discovered through the exercise of reasonable care. C. The law school reasonably did not expect that a young child would be brought to the lecture on an abstruse legal topic. D. The chair in question had not previously caused problems and was not known to be defective.

The correct answer is B.

Uniform Comparative Fault Act provides: Section 1. [Effect of Contributory Fault] (a) In an action based on fault seeking to recover damages for injury or death to person or harm to property, any contributory fault chargeable to the claimant diminishes proportionately the amount awarded as compensatory damages for an injury attributable to the claimant's contributory fault, but does not bar recovery. This rule applies whether or not under prior law the claimant's contributory fault constituted a defense or was disregarded under applicable legal doctrines, such as last clear chance. (b) "Fault" includes acts or omissions that are in any measure negligent or reckless toward the person or property of the actor or others, or that subject a person to strict tort liability. The term also includes breach of warranty, unreasonable assumption of risk not constituting an enforceable express consent, misuse of a product for which the defendant otherwise would be liable, and unreasonable failure to avoid an injury or to mitigate damages. Legal requirements of causal relation apply both to fault as the basis for liability and to contributory fault. Section 2. [Apportionment of Damages] .... (b) In determining the percentages of fault, the trier of fact shall consider both the nature of the conduct of each party at fault and the extent of the causal relation between the conduct and the damages claimed. .... Assume that A is injured while using a product that was defectively manufactured by B and sold at retail by C. A was in no way responsible for the accident caused by the defective product, but after the accident unreasonably neglected to go to a doctor, which aggravated the injuries. In all, A's losses totaled up to $90,000. If a jury finds that A, B, and C were equally at fault, what is the maximum that A can recover from B and C? (Ignore the issue of whether the recoverable damages, if any, will be paid by A, by B, or in part by both.) A. $90,000 B. $60,000 C. $30,000 D. Zero

The correct answer is B.

An independent contractor negligently installed fixed seating in a classroom by failing to put nuts on two of the four mounting bolts. The school thereafter negligently failed to discover the error, although it could have done so through the exercise of reasonable care. As a result of the combined negligence of the independent contractor and the school, a seat broke loose from its mounting bracket and a student fell to the floor, suffering injuries. Assume (a) that the student sues both the independent contractor and the school for negligence; (b) that the jurisdiction follows standard common law principles governing the vicarious liability of employers; (c) that the jury finds that the defendants were equally responsible for the accident; (d) that the student was not at fault and suffered $100,000 in damages; and (e) a state statute contains the following language: A state statute in the applicable jurisdiction provides in relevant part: § 33.001. Proportionate Responsibility This chapter applies to tort actions based on negligence, recklessness, or strict liability. A claimant may not recover damages if his percentage of responsibility is greater than 50 percent. §33.012. Amount of Recovery (a) If the claimant is not barred from recovery under Section 33.001, the court shall reduce the amount of damages to be recovered by the claimant with respect to a cause of action by a percentage equal to the claimant's percentage of responsibility. §33.013. Amount of Liability (a) Except as provided in Subsection (b), a liable defendant is liable to a claimant only for the percentage of the damages found by the trier of fact equal to that defendant's percentage of responsibility with respect to the personal injury, property damage, death, or other harm for which the damages are allowed. (b) Notwithstanding Subsection (a), each liable defendant is, in addition to his liability under Subsection (a), jointly and severally liable for the damages recoverable by the claimant under Section 33.012 with respect to a cause of action if: (1) the percentage of responsibility attributed to the defendant with respect to a cause of action is greater than 50 percent.... The most that the student can collect from the school is: (A) Zero. (B) $50,000. (C) $75,000 (D) $100,000

The correct answer is B. (The school is not vicariously liable for the work of the independent contractor. Under the statute, liability is several, because the school's contribution was not greater than 50%.)

A sport-utility vehicle with a roof rack was manufactured by a company called Maker. The SUV was then sold to a customer, named Buyer, by a car dealer, called Dealer, which sells vehicles manufactured by Maker. While the SUV was traveling down the highway part of the roof rack detached. The loose part first bounced on the roof of Buyer's SUV, denting the metal and chipping the paint. The loose part then struck a small car driven by another driver, called Other, shattering the windshield of Other's car, but not injuring the passengers. Assume that Other sues Dealer seeking to impose liability for the broken windshield under Restatement, Second, of Torts §402A in a jurisdiction which has not judicially or legislatively modified the terms of liability expressed in the Restatement. If Other establishes that the roof rack was defectively manufactured by Maker, Dealer is most likely to avoid liability by: A. Arguing that Other did not buy the SUV from Dealer. B. Arguing that the incident caused only property damage, not personal injuries, to Other. C. Arguing that Other was not a "user or consumer" of the defective product. D. Establishing that Dealer was unaware that the rack was defective.

The correct answer is C.

A young boy, Millo, age 11, was a member of the church choir. The director of the choir was an ordained church minister under whose direction the choir practiced. There was a close relationship of trust and confidence between Millo and the choir director, and Millo thought of him as a "second father." One day, after choir practice, Millo was helping the minister to put away the chairs. No one else was around. The minister took advantage of the occasion to sexually molest the child. As a result of the incident, Millo later suffered severe emotional distress, as did Millo's parents when they found out about the incident from the child six months later. Assume that prior to the sexual molestation of Millo, Millo's parents had heard rumors that the minister had engaged in similar sexual misconduct in the recent past with other boys in the choir. Assume further that the jurisdiction follows the common law rule that contributory negligence is a total, rather than comparative, defense to an action based on negligence. If Millo sues the minister for battery and the parents bring a derivative loss of consortium claim, a finding of carelessness on the part of the parents will: A. Bar the parent's claim, but not bar Millo's claim. B. Bar both the parent's claim and Millo's claim. C. Bar neither the parent's claim nor Millo's claim. D. Reduce the parent's recovery, but not affect Millo's claim.

The correct answer is C.

A pregnant woman, Sybil, visited the apartment of her friend, Delphia, to celebrate Delphia's birthday. During the visit, Sybil was electrocuted when she flipped the light switch in the powder room, which had recently been replaced by Delphia's landlord. Delphia was unaware that the switch was dangerous, but could easily have discovered that fact if she had checked to see if the work had been properly performed by the landlord. The jolt of electrical current was so severe that Sybil went into labor. The fetus Sybil was carrying was stillborn. The fetus was at such an early stage of development that it would not have been capable of surviving outside the womb. If Sybil sues Delphia on her own behalf for negligence in a jurisdiction adhering to the traditional categories on premises liability, the court will likely enter judgment for: A. Sybil, because the risk of electrocution makes electricity an abnormally dangerous activity. B. Delphia, if she had no ownership interest in the apartment. C. Sybil, if Delphia could have discovered the dangerous condition of the switch through the exercise of reasonable care. D. Delphia, because Delphia was unaware that the light switch was dangerous.

The correct answer is D.

A young boy, Millo, age 11, was a member of the church choir. The director of the choir was an ordained church minister under whose direction the choir practiced. There was a close relationship of trust and confidence between Millo and the choir director, and Millo thought of him as a "second father." One day, after choir practice, Millo was helping the minister to put away the chairs. No one else was around. The minister took advantage of the occasion to sexually molest the child. As a result of the incident, Millo later suffered severe emotional distress, as did Millo's parents when they found out about the incident from the child six months later. If Millo seeks to hold the minister liable for breach of fiduciary duty, the most likely result is judgment for: A. The minister, because there was no fiduciary relationship between the minister and Millo. B. Millo, because conduct that causes injury is a violation of fiduciary duty. C. The minister, because breach of fiduciary duty requires evidence of culpability. D. Millo, because the minister violated the trust that had been reposed in him.

The correct answer is D.

ABC Manufacturing Co. manufactured and sold a high-speed printing press to XYZ Printers, by whom Robert is employed. The press includes a circular plate cylinder that spins at a very high speed. On occasion a foreign object, known in the trade as a "hickie," finds its way onto the plate of the unit, causing a blemish or imperfection on the printed page. To remove a "hickie," ABC knows that it is customary practice for an employee to apply a piece of plastic to the printing plate while it is spinning. Robert performed this practice, known as "chasing the hickie," and while doing so suffered serious injuries to his hand. All employees, including Robert, knew that "chasing the hickie" was a dangerous procedure. An expert testifies that a safety-guard at the point of operation, which could have prevented Robert's injury, was both technologically and economically feasible and is utilized in similar machinery without causing difficulty. If Robert sues ABC on a defective-design strict liability theory in a jurisdiction adhering to pure comparative negligence, the most likely result is that: A. The action will be barred if Robert's injury is covered by worker's compensation insurance. B. The action will be barred because the danger was open and obvious, C. The action will barred by implied secondary assumption of the risk. D. Robert will recover at least part of his damages.

The correct answer is D.

During a dinner party, the host gives a 16-year-old child a glass of wine with the express permission of the child's parents who are present. The child becomes "tipsy" and later falls down the stairs of the host's home. If a suit is brought on behalf of the child against the host based on negligence in a pure comparative fault jurisdiction, the most likely result is judgment for: A. The host, if the jurisdiction follows the common law rules on social host liability. B. The host, because the parents' express consent is a complete defense. C. The host, because an adult who provides alcohol to a minor is strictly liable for resulting injuries. D. The child, if the jury finds that the host's conduct was unreasonable.

The correct answer is D.

Andrea, age four, suffered serious burns when she got out of bed one night to go to the bathroom and tripped on the electric cord connected to a hot water vaporizer. When Andrea tripped on the cord the vaporizer separated into its three component parts-a large, wide-mouthed glass jar, a metal pan, and a plastic top-heating unit. When the plastic top-heating unit came off, the hot water in the glass jar poured out, causing Andrea's burns. In an action alleging defective design of the vaporizer, Andrea's representative presents evidence that it was practically feasible at the time of sale by the manufacturer to design a vaporizer that would secure the heating unit to the water receptacle so that water would not escape when the vaporizer tipped over. Under the Restatement, Third, of Torts: Products Liability, in deciding whether the product as designed was not reasonably safe, which of the following considerations is irrelevant: (1) the foreseeability that the vaporizer might be accidentally tipped over; (2) consumer knowledge or lack thereof that the water in the glass jar is scalding hot; (3) the added cost of the safer alternative design; and (4) the relative convenience of a vaporizer with a lift-off cap. A. 1 B. 2 C. 3 D. 4 E. 1, 2, 3, and 4 are all relevant.

The correct answer is E.

Tort liability based on ordinary negligence can be discharged (extinguished) by the tortfeasor's filing bankruptcy.

True

An insane person can be held liable for committing an intentional tort.

True

Conduct based on a mistake of fact induced by the plaintiff ordinarily does not give rise to tort liability.

True

Like classic contributory negligence, the rule of sovereign immunity, when it applies to a case, is a total defense to holding the defendant liable for a tort.

True

Negligence and recklessness both involve failure to exercise care

True


Conjuntos de estudio relacionados

The Nature and Conservation of Energy

View Set

The Enlightenment Assignment and Quiz

View Set

Medical Terminology Chapter 20 Vocabulary

View Set

Supply Chain Management: Demand Planning: Forecasting and Demand Management (CH 12)

View Set